Easy Tutorial
For Competitive Exams

GMAT Analytical Writing Assessment

  • 1.An Argument

    "The recent surge in violence in the southern part of the city is a result of a shortage of police officers and an absence of leadership on the part of the city council. In order to rectify the burgeoning growth of crime that threatens the community, the city council must address this issue seriously. Instead of spending time on peripheral issues such as education quality, community vitality, and job opportunity, the city council must realize that the crime issue is serious and double the police force, even if this action requires budget cuts from other city programs."

    Discuss how well reasoned . . . etc.

    Analysis

    In the argument above, the author concludes that the city council is not doing its job well and needs to focus on expanding significantly the police force in order to combat recent growth in the level of crime. The premise of the argument is that crime is expanding while the city council focuses on ostensibly unrelated matters such as education reform. However, the argument is flawed because it falsely assumes that the city council’s efforts to improve quality of life are entirely unrelated to levels of violence and it assumes that the crime problem can be solved by merely increasing the police force.

    First, the argument wrongly assumes that issues of educational opportunity, community vitality, and job availability have no bearing on crime. However, the author fails to support this assumption. It is entirely possible that the crime level spiked due to a recent and sizeable layoff at a major nearby factory that pushed countless citizens out of work and onto the streets. With individuals struggling to survive, it should come as no surprise that people are turning to crime.

    Second, the reasoning in the editorial is flawed because it erroneously assumes that increasing the police force will directly address the root of the crime problem and reduce the level of crime. Yet, a landmark study published in early 2008 showed that increasing the size of a police force beyond a certain point provides extremely small marginal returns in the reduction of crime. Given the fact that the local police force is already above this threshold, the editorial’s author wrongly assumed that a doubling of the police force will materially decrease the crime rate.

    Moreover, the argument could be improved by appealing to the city’s history with fighting crime and managing the size of its police force. In particular, approximately 25 years ago, the city council faced a situation very similar to the one it faces today: a rising crime rate and growing spending on community development. The city council decided to increase the size of its afterschool programs’ budget by about 75% and this reduced crime dramatically. Faced with the same situation today, the city council should follow the path it took 25 years ago.

    In conclusion, the argument in the newspaper editorial is flawed because it assumes that educational opportunity, job availability, and community vitality are not related to the level of crime a community experiences. Moreover, the argument wrongly concludes that an increase in the police force will address the root issue behind the crime, which the argument assumes is an inadequate number of police officers.

  • 2.An Argument

    The following appeared in a memorandum from the director of research and development at Ready-to-Ware, a software engineering firm.

    The package of benefits and incentives that Ready-to-Ware offers to professional staff is too costly. Our quarterly profits have declined since the package was introduced two years ago, at the time of our incorporation. Moreover, the package had little positive effect, as we have had only marginal success in recruiting and training high-quality professional staff. To become more profitable again, Ready-to-Ware should, therefore, offer the reduced benefits package that was in place two years ago and use the savings to fund our current research and development initiatives.

    Discuss how well reasoned you find this argument. In your discussion be sure to analyze the line of reasoning and the use of evidence in the argument. For example, you may need to consider what questionable assumptions underlie the thinking and what alternative explanations or counterexamples might weaken the conclusion. You can also discuss what sort of evidence would strengthen or refute the argument, what changes in the argument would make it more logically sound, and what, if anything, would help you better evaluate its conclusion.

    Analysis

    The argument presented by the Ready-to-Ware company has some validity in theory, however it lacks key information that would assist in identifying if the reasoning is truly valid. Their stance is that the current benefits package is too expensive and is a possible reason for a decline in company profits. Additionally, they believe the benefits package is unsuccessful in recruiting and training high-quality staff and that by offering a reduced package, they can increase profitability by reallocating funds to their research and development. There are a few key assumptions they are making in their argument that require additional information to be validated. Without this additional information, this argument rests solely on assumption and not fact.

    The first key flaw with this argument is around the idea of their decline in profitability. The argument heavily implies that the benefits and incentives that is currently offered is a reason for the company’s decline in profits. Without any factual evidence to back this claim up, it simply cannot be taken as a fact. There are many reasons why the Ready-to-Ware company may have declining profits. These could include rising costs of hard goods, higher competition, a change in leadership, and many other factors. A few questions that would be helpful to have answered are the following. Does your competition have a similar benefits and incentives package, and if so, are they able to maintain profitability? Was there a loss in revenue over the last two years? Was there an increase in hard costs, regulatory costs, or any other type of cost that could be impacting the bottom line of the company? And per employee, what is the cost to the company of offering better benefits and incentives? If those questions were answered we could draw a better conclusion in regards to the relationship between the company’s profitability and the introduction of the current benefits and incentives package.

    The second key flaw with this argument is around the assumption that reallocating funds from the benefits and incentives package into research and development will result in increased profitability for the company. There is no evidence presented that a current lack in research and development funds is holding the company back from maintaining profit margins. It is very possible that research and development would, in fact, help increase profitability. However, it is equally as likely that the existing research and development team does not have the right employees leading their projects and an increase in funds in their department would not yield a significant gain to the company. One would need to answer some key questions in order to draw a direct conclusion from this assumption. These include the following. Has the research and development team been lacking funds in recent years? Is research and development accepted as a key area for increased revenue in this industry? Is the current research and development team staffed appropriately? Do other research and development teams offer very attractive benefits and incentives programs? Without answering those key questions, this portion of the overall argument relies completely on assumption and not fact.

    The third key flaw with the argument is in regards to the history of the benefits and incentives packages. Without understanding why the change was made to a more attractive program two years ago, one cannot assume that their previous program was acceptable. It is very possible that their previous benefits and incentives program was a detriment to the company and resulted in employees leaving for competitors who offered better programs. This could have resulted in a turnover period that required the company to hire and train new staff, potentially impacting profitability. Without any additional information in, the argument presented is simply an assumption. One would need to to answer many questions regarding the previous plan in order to draw a final conclusion. These questions would include: What was the reason for the change to the benefits and incentives program? What types of programs do your competitors offer? Was Ready-to-Ware losing employees because of the benefits package two years ago? Without answers to these questions, we are left with assumptions, and cannot fully judge the argument.

    While on the surface this argument may make some logical sense, it lacks a great deal of information that is integral to identifying if this is a valid argument. The company certainly needs to make some type of change in order to regain profitability, however the benefits and incentives program could very well have nothing to do with the decline in profitability. The additional questions that have been presented in my analysis would allow us to gain a greater understanding of the facts, and thus make a better decision regarding whether or not the actions proposed would solve the profitability problem. Until those questions are answered, this argument rests solely on assumption.

  • 3.An Argument

    The following appeared in an announcement issued by the publisher of The Mercury, a weekly newspaper:

    "Since a competing lower-priced newspaper, The Bugle, was started five years ago, The Mercury’s circulation has declined by 10,000 readers. The best way to get more people to read The Mercury is to reduce its price below that of The Bugle, at least until circulation increases to former levels. The increased circulation of The Mercury will attract more businesses to buy advertising space in the paper."

    Discuss how well reasoned . . . etc.

    Analysis

    The argument claims that by reducing its price below that of the The Bugle, The Mercury will gain back it’s readers and attract more businesses to buy ad space in the paper. This argument relies on assumptions that have absolutely no clear evidence. We will discuss several of these assumptions including, that the decline in The Mercury’s circulation was directly due to The Bugle. That a lower price for the Mercury will result in an increase in circulation and finally that businesses will buy more ad space with increased circulation of The Mercury. Without clear evidence and more information, this argument has no feet to stand on.

    First, the argument readily assumes that a decline in the Mercury’s circulation was due to the start up of The Bugle five years ago. The argument fails to provide any evidence for this claim. Perhaps the decline in circulation was due to a decrease in the quality of the newspaper. Maybe readers prefer to read the Mercury online and therefore do not require the paper form anymore. The argument could have been much clearer if it had showed a direct correlation between The Bugle and the decrease in circulation for The Mercury.

    Secondly, the argument claims that lowering the price of The Mercury below that of The Bugle will result in circulation increasing. This is again very weak and unsupported. There is no guarantee that lowering the price will increase the circulation. Also, lowering the price may not be an option for The Mercury. Lowering the price may lower their profit margins so much that they are unable to keep the paper in business. They would have to do a cost/benefit analysis to determine if this was even a viable option. However, without a guarantee that it would increase circulation, it would be a futile strategy. The argument is very unconvincing and does not provide the necessary details or evidence to give it any strength

    Finally, the argument assumes that businesses will buy advertising space once the Mercury’s circulation increases. This argument tells us that businesses only make their ad space decisions based on the circulation of the paper. This is a weak and flawed assumption. Businesses may take many things into account before making a decision on which paper to advertise in. For example, when I owned my M&M Meat franchise, we made our advertising decisions based on many things. What demographic were we reaching? What was the cost of the ad space? How reliable is the distribution system? These were just a few of the decisions we faced. The argument in the passage would have been much more convincing had it provided more evidence regarding the businesses decision process.

    The argument is weak, flawed and unconvincing. It fails to provide evidence to support the argument that a decline in the Mercury’s circulation was due to the start up of The Bugle five years ago. The claim could have been strengthened had it showed us a direct correlation between the two papers. Also, if it would have shown some evidence that a business would make its advertising decisions based solely on circulation. Without this information, the argument remains unsubstantiated and open to debate.

  • 4.An Argument

    "Ronnie’s Auto Body Shop commenced business four months ago at the location formerly occupied by the Mystique Beauty Parlor. Ronnie’s Auto must be doing well at this location, because it intends to open a big body shop in an adjacent town. Mystique, on the other hand, has seen a lower volume of business in its first year at its new location compared to the prior year at its former location. Mystique definitely erred in shifting to its new location; its former location is a better site.

    Discuss how well reasoned you find this argument. In your discussion be sure to analyze the line of reasoning and the use of evidence in the argument. For example, you may need to consider what questionable assumptions underlie the thinking and what alternative explanations or counterexamples might weaken the conclusion. You can also discuss what sort of evidence would strengthen or refute the argument, what changes in the argument would make it more logically sound, and what, if anything, would help you better evaluate its conclusion.

    Analysis

    The author offers a flawed argument in claiming that Mystique erred in shifting locations and that its former location is a better site. The author offers insufficient evidence for these claims while providing an illogical comparison. In particular, the author assumes that Ronnie’s Auto Body Shop has been successful in Mystique’s former location. Second, the author overlooks the differences in the two business types. Third, the author inappropriately equates a lower volume of business to a poor decision by Mystique regarding location. Let’s elaborate on each of these issues in turn.

    One problem in the argument is that the author assumes that Ronnie’s Auto is doing well at its current location. The only evidence that the author offers in support of this claim is that Ronnie’s Auto intends to open a big body shop in an adjacent town. But there are many possible reasons as to why Ronnie’s Auto plans to open a large shop in another town. Most notably, perhaps the existing location is performing poorly and Ronnie’s Auto plans to close this location after opening up elsewhere.

    A second problem in the argument is the apples-to-oranges comparison between an auto body shop and a beauty parlor. The customer base for the two businesses is probably very different. Even if Ronnie’s Auto is doing well at its current location, a totally different type of business with a totally different customer base would not necessarily do well at the same location. Perhaps Ronnie’s Auto is located in a commercial area with similar types of businesses (e.g. other auto repair and parts businesses). Mystique Beauty Parlor is unlikely to thrive in such a location, since its customers would probably not want to receive beauty treatments in such an area.

    Finally, a third problem is that the author concludes based solely on “lower volume of business” that Mystique erred in shifting locations. But how much lower is Mystique’s volume of business in the new location? Perhaps the drop in business volume is very small. In fact, we would expect some drop when moving to a new location, as it takes time to reestablish a customer base. The new location may hold much greater long-term promise, and future business volume may be much higher. We also do not know what volume of business really means. By volume, does the author mean customers or revenue? Either way, a lower volume of business does not necessarily mean lower profits. Perhaps Mystique’s lease and other costs are lower at the new location, resulting in higher profits. If so, we cannot claim that Mystique make a mistake in moving. But even if the move did result in lower profits, the move was not a mistake unless Mystique’s owners have this as a primary concern. Perhaps profit is secondary to Mystique’s owners, and they moved the beauty parlor to a location that affords them other benefits such as a better quality of life.

    In conclusion, the author’s claims that Mystique erred in shifting locations and that its former location is better are unsupported. As discussed above, there are far too many unknown variables for the author to make such a definite conclusion. Had the author considered the points discussed above, the argument could have been presented more thoughtfully.

  • 5.An Argument

    The following appeared as part of an annual report sent to stockholders by Olympic Foods, a processor of frozen foods. "Over time, the costs of processing go down because as organizations learn how to do things better, they become more efficient. In color film processing, for example, the cost of a 3-by-5-inch print fell from 50 cents for five-day service in 1970 to 20 cents for one-day service in 1984. The same principle applies to the processing of food. And since Olympic Foods will soon celebrate its twenty-fifth birthday, we can expect that our long experience will enable us to minimize costs and thus maximize profits".

    Analysis

    The author concluded that with long experience of 25 years, Olympic foods will be able to maximize profits and minimize costs because the processing cost has gone down in color film industry. The line of reasoning is that the same approach in film processing industry should be applicable to the other industries. This argument is not sound, however, because it is not necessary that same thing would happen to food industry. It depends upon many other factors.

    Firstly, the argument assumed that the color film industry is similar to food industry. One must not forget that an Olympic food is an industry for frozen foods or perishable products. These products require fast transportation and special equipments in order to keep fresh or the entire stock will get junked. These requirements can claim substantial costs and it is very likely that they can never be cut. One the other hand, color film is a consumer product which stays much longer and is not perishable. Therefore, it is possible that the cost-cutting approach is not applicable to the food industry.

    Secondly, the author failed to address other factors that are important to a company’s success. It is well known that in the long run maximization of profits occurs due to low cost of production. But it’s not the only factor they consider. Other factors such as demand for the product, selling price, and overall competition in the market should also be taken into consideration. Today, buyers become the king in the market. If other companies’ products are available at lower price with same quality or at similar price with higher quality, then people don’t buy the Olympic’s product. Therefore, if most consumers choose other companies’ products, then the objects of higher profits and lower cost can’t be attained.

    Finally, the speaker did not include any information on Olympic management approach. Rather, it just mentioned the long experience of 25 years in food industry. While there is rough correlation between long experience and ability to maximize profit, it is not always the case. If the Olympic fails to accumulate valuable management experience, such as time-consuming strategic alliance, learning from failure, etc, then Olympic long experience will not enable it to minimize costs and thus maximize profits

    In sum, the argument is not compelling because it omitted many other factors that must be addressed in order to make proper conclusion. If the author has considered the difference between color film and food industry, selling price, and product quality, the argument would be more convincing.

  • 6.An Argument

    The following appeared in a memorandum from the business department of the Apogee Company:

    When the Apogee Company had all its operations in one location, it was more profitable than it is today. Therefore, the Apogee Company should close down its field offices and conduct all its operations from a single location. Such centralization would improve profitability by cutting costs and helping the company maintains better supervision of all employees.

    Discuss how well reasoned you find this argument. In your discussion be sure to analyze the line of reasoning and the use of evidence in the argument. For example, you may need to consider what questionable assumptions underlie the thinking and what alternative explanations or counterexamples might weaken the conclusion. You can also discuss what sort of evidence would strengthen or refute the argument, what changes in the argument would make it more logically sound, and what, if anything, would help you better evaluate its conclusion.

    Analysis

    The argument claimed by business department of the Apogee Company in a memorandum that centralization of the company and closing down its field offices is necessary to maximize the company’s profits and minimize the costs. They have wrongly used the centralization concept. Can a company that has expanded through a wide network of field offices can suddenly shut down just because it has failed to raise profits. Even though their claim may well have a merit, the business department presents a poorly and weak reasoned argument, based on several questionable premises and assumptions, and based solely on the evidence that it offers, we cannot accept their argument as valid.

    The primary issue with the business department’s reasoning lies in their unsubstantiated premises. Some field offices may be profitable and some may be in loss making. The ones which are profitable cannot be shut down. Also instead of closing down the offices, it is better to analyze the problems with respect to internal and external. Internal problems such as corrupt or rigid management, inefficient workers, large attrition rate or sudden hiring rate, difficult working conditions. External problems such as any force majeure incidents such as flood, drought, earthquake, etc, poor customer’s response or competitor’s good hold on the market. Depending on the quantum of problem, it is better to list down the effective strategy with a set target date for the particular loss making office.

    The secondary issue in the argument lies in the cost-benefit analysis. The business department has failed to explicitly mention that whether proper pros and cons were considered in the cost sheet. Initially, the cost benefit may seem to be high. Local field offices would have been much effective in handling local customers and vendors. If centralization of the company is adopted, then the employers have to travel for longer distances for business meetings, for providing services to the customers or for inspection at manufacturer’s works .Certainly, the cost will be higher and thus minimizing the profits as expected initially. The business development’s premises, the basis for their argument, lack any legitimate evidentiary support and render their conclusion as unacceptable.

    The tertiary issue in the argument is high probability of getting a bad publicity. Simply closing the field offices for the process of cost-cutting will garner negative publicity in the market. The benefit will go to its competitors. Hence, the company will fall into a more vicious circle of reduced profit and bad reputation in the market. Hence, the business development weaken their argument by making assumptions and failing to provide explication of the links between increasing profits and closing down the field offices for centralization, that the department assume to be exists.

    While the business department has several key issues in its argument’s premises and assumptions, that is not to say the entire argument is without base. The departments’ main objective was to identify the main area of loss in the company. If centralization was the only solution, the business department could have given more statistical data. It could have explained that centralized management would take the ultimatum decisions and could have provided an efficient business strategy. They could have suggested ways to handle the public attention for the closure of local field offices.

    In sum, the business development’s illogical argument is based on unsupported premises and unsubstantiated assumptions that render their conclusion as invalid. They have overlooked the above addressed points. If the business development truly hopes to change its readers’ mind on the issue, they would have to largely restructure their argument, fix the flaws in their logic, clearly explicate their assumptions, and provide evidentiary support. Without, all these things, their poorly reasoned argument will likely convince few people.

  • 7.An Argument

    The following appeared in a memorandum issued by a large city’s council on the arts.

    "In a recent citywide poll, fifteen percent more residents said that they watch television programs about the visual arts than was the case in a poll conducted five years ago. During these past five years, the number of people visiting our city’s art museums has increased by a similar percentage. Since the corporate funding that supports public television, where most of the visual arts programs appear, is now being threatened with severe cuts, we can expect that attendance at our city’s art museums will also start to decrease. Thus some of the city’s funds for supporting the arts should be reallocated to public television."

    Discuss how well reasoned . . . Etc.

    Analysis

    First of all, the author commits the “Confused Cause and Effect” fallacy. The argument depends on the assumption that increased exposure to the visual arts on public television has caused a similar increase in local art-museum attendance in the past years. However, the poll that increased art-museum attendance is statistically correlated with similar increases in television viewing of visual-arts programs, does not necessarily mean that the increased television viewing of arts is the cause of the rise in museum attendance. There may be other factors relevant to increased interest in the local art museum during the past years. For example, some larger social or cultural factors may cause greater public interest in municipal art museums.

    Second, the argument does not address the effectiveness of citywide poll conducted five years ago. The survey may be biased. If the respondents do not properly represent the whole residents, then the poll is not convincing. Moreover, since the survey was conducted five years ago, the statistics can become invalid and can no longer be used as future prediction.

    In conclusion, the argument is not convincing enough and would be strengthened if the author were to eliminate other significant factors that might have caused the increase in visits to the local art museum, as well as to address the soundness o the survey conducted five years ago.

  • 8.An Argument

    The following appeared in a report presented for discussion at a meeting of the directors of a company that manufactures parts for heavy machinery. “The falling revenues that the company is experiencing coincide with delays in manufacturing. These delays, in turn, are due in large part to poor planning in purchasing metals. Consider further that the manager of the department that handles purchasing of raw materials has an excellent background in general business, psychology, and sociology, but knows little about the properties of metals. The company should, therefore, move the purchasing manager to the sales department and bring in a scientist from the research division to be manager of the purchasing department."

    Discuss how well reasoned . . . Etc.

    Analysis

    The author argued that the company should replace the current manager with a scientist from the research division as the manager of the purchasing department as a result of falling revenues. The argument is based on the two facts: 1) The company revenues fall at the same as do delays in manufacturing; and 2) The current manager who is responsible for purchasing of raw materials knows little about the properties of metals. The line of reasoning is not sound and, therefore the conclusion is not compelling.

    Firstly, the argument failed to address the casual relationship between falling revenues and delays in manufacturing. The fact that the falling revenues that the company is experiencing coincide with delays in manufacturing does not mean one causes another. It may be other reasons that cause the company to experience the falling revenues. For example, the quality of products does not satisfy its customers, and as a result, many previous clients are turning to its competitors.

    Furthermore, it is not reasonable to conclude that a scientist can save the poor planning in purchasing metals. If the scientist is skilled in the properties of metals but poor at planning, the delays in manufacturing may be worse. The purchasing manager must know where he could find those kinds of metals, what prices they are, and when they can be transported to his company. In fact, a department manager in business background is really acceptable. Because when he bumps against a technical problem, he can easily seek advice from the engineer team in manufacturing department.

    In conclusion, the delays in manufacturing were not necessarily caused by the purchasing manager's poor knowledge about the properties of metals. Maybe the company must replace the purchasing manager; however, a scientist may also not be competent for this job. It must find someone who really knows how to plan and how to buy qualified metals in time.

  • 9.An Argument

    The following appeared as part of an article in a magazine devoted to regional life.
    "Corporations should look to the city of Helios when seeking new business opportunities or a new location. Even in the recent recession, Helios’s unemployment rate was lower than the regional average. It is the industrial center of the region, and historically it has provided more than its share of the region’s manufacturing jobs. In addition, Helios is attempting to expand its economic base by attracting companies that focus on research and development of innovative technologies."
    Discuss how well reasoned . . . Etc.

    Analysis

    In this article, corporations are encouraged to settle their business in the city of Helios when seeking new business opportunities or a new location. The recommendation is based on the fact that Helios is the industrial center of the region, given that the city provided more than its share of manufacturing jobs and its unemployment rate is lower than average. Furthermore, the city is taking efforts to expand the economic base of the city by attracting companies that focus on research and development of innovative technologies. This argument is somewhat persuasive but not convincing for the following two reasons.

    First, lower-than-average unemployment rate seems attractive to job applicants, but actually not good for corporations. Because of the lower than average unemployment rate in Helios, companies that should recruit large numbers of employees would not find Helios a great place. Lower unemployment rate means that the incoming corporations will have to recruit new workers outside the city of Helios or lure the existing workers away from their current jobs with higher wages. Both of these two alternatives will result in higher recruiting costs, and therefore make Helios a BAD place for business.

    Second, it is questionable whether the available labor in Helios could support all types of business. Even though the city of Helios has great environment for industrial and manufacturing companies, it is likely that the local prospective employees are not suitable for other types of corporations, such as financial service companies. Furthermore, the city’s attempt to attract companies that focus on research and development of innovative technologies does not necessarily result in the expanding economic base as it expect.

    In sum, the author did not provide compelling reasons for why Helios is a great site for a company wishing to relocate. Nor has the author provided compelling reasons for companies seeking new business opportunities to choose Helios. If the author had included the above items, it may be more convincing.

  • 10.An Argument

    The following appeared in the health section of a magazine on trends and lifestyles.
    "People who use the artificial sweetener aspartame are better off consuming sugar, since aspartame can actually contribute to weight gain rather than weight loss. For example, high levels of aspartame have been shown to trigger a craving for food by depleting the brain of a chemical that registers satiety, or the sense of being full. Furthermore, studies suggest that sugars, if consumed after at least 45 minutes of continuous exercise, actually enhance the body’s ability to burn fat. Consequently, those who drink aspartame-sweetened juices after exercise will also lose this calorie-burning benefit. Thus it appears that people consuming aspartame rather than sugar are unlikely to achieve their dietary goals."
    Discuss how well reasoned . . . Etc.

    Analysis

    The article concludes that it is better to use sugar than the artificial sweetener aspartame for those people who want to lost weight. The author uses two reasons to support his viewpoints. First, high levels aspartame will cause people to eat more. Second, sugar can help fat-burning if used properly. These reasons can only tell reader that aspartame has some bad side-effect while sugar has some good effect, but they can't prove that sugar is better than aspartame for weight-control. Therefore, the reasoning is less than sufficient and the argument is not convincing.

    In the first reason, author fails to define what the level of amount is “high”. The normal dose of aspartame that people consuming may be very low. If most people use artificial sweetener that contains lower level of aspartame than the one that will trigger a craving for food, then the advantage of consuming sugar no long exists. Moreover, the author didn't mention whether sugar will also contain that chemical. If it is the case, people should turn to the third product.

    The second reason is also not sound, because it requires people not to drink until 45 minutes after exercise. This procedure is difficult to follow because most people drink juices immediately after or during excise. If they have to wait for 45 minutes, they may not need drink any longer. Also, the author didn't mention whether the calorie that sugar itself generates is less than the amount it helps to burn. Finally, the article fails to address whether the aspartame's side-effect, if any, is bigger than sugar’s.

    In conclusion, the argument is nor sound or convincing. However, if it had included the above mentioned items, it would be more compelling.

  • 11.An Argument

    The following appeared in the editorial section of a corporate newsletter.
    "The common notion that workers are generally apathetic about management issues is false, or at least outdated: a recently published survey indicates that 79 percent of the nearly 1,200 workers who responded to survey questionnaires expressed a high level of interest in the topics of corporate restructuring and redesign of benefits programs."
    Discuss how well reasoned . . . Etc.

    Analysis

    In this argument, the arguer claims that workers are becoming more and more enthusiastic about management issues. To substantiate this conclusion, the arguer provides the result of a survey, which point out that more than half of the workers asked were considerably interested in the topics of certain corp9orate programs, While this argument has several merits, there are some logical errors that seriously undermine the reasoning.

    In the first place, the procedure of the sampling is unclear. With different views on management issues, the 1,200 workers should come from different fields. If they worked in the same department, however, the results of the survey would be unreliable. In addition, the arguer must provide a more detailed process of the questionnaire.

    In the second place, no evidence is offered to indicate that most of the other workers take the same level of interest in those programs as the ones asked, for the reason that the 1,200 workers are not representative of all of the workers. Hence, the arguer must present much more data such as the total number of worker.

    Last but not least, the arguer makes a hasty generalization about the types of issues that the workers are interested in. It is entirely possible, for example, that most of the 1,200 workers were concerned about only the corporate restructuring and redesign of benefits programs and still apathetic about other issues. If so n the survey on which the argument relies would lack credulity and therefore would not lend strong support to the argument.

    In conclusion, the arguer fails to demonstrate that the results of the survey are reasonable and convincing. To solidify the argument, the arguer must present more concrete information about the procedure and many other samples of the survey. Furthermore, the arguer would have to provide a clearer definition of management issues so that the workers might have more choices when answering the questions of the survey.

  • 12.An Argument

    The following appeared in the opinion column of a financial magazine.
    On average, middle-aged consumers devote 39 percent of their retail expenditure to department store products and services, while for younger consumers the average is only 25 percent. Since the number of middle-aged people will increase dramatically within the next decade, department stores can expect retail sales to increase significantly during that period. Furthermore, to take advantage of the trend, these stores should begin to replace some of those products intended to attract the younger consumer with products intended to attract the middle-aged consumer.
    Discuss how well reasoned . . . Etc.

    Analysis

    The speaker concluded that department stores should replace some of their products intended to attract the younger consumer with those intended to attract the middle-aged consumer. The argument is based on the following two facts: 1) the middle-age consumers devote more percent of their retail expenditure to department store than do younger consumers; and 2) the number of middle-aged people will increase dramatically in the next decade. The above argument omits some paramount items that must be involved in the reasoning process; therefore, it does not constitute a logical argument in favor of the conclusion and it certainly is not persuasive and sound.

    First, it is questionable that middle-aged consumers will increase greatly within the next decade. According to the above argument, one can not get the conclusion that the middle-aged consumers will increase during the next decade. The population of the present young people should be presented. If now the number of younger people surpasses the number of middle-aged people greatly, then one can get the above conclusion. If not, the conclusion is wrong. The above article gives its reader the impression that middle-aged customers will increase just because the young people will become middle-aged patronages in ten years. So, it is illogical.

    Second, the author assumes that the average expenditure to department store products and service of middle-aged customers is greater than those of young ones. In fact, young people usually spend more money than older ones. So, if the author wants to convince others that the opposite is right, he must present related information or data collected in some surveys.

    In summary, the article leaves out the paramount issues mentioned above. Hence it is not thorough or convincing. If it had included persuasive information that could eliminate its readers' doubts, it would have been more sound and compelling.

  • 13.An Argument

    The following appeared in the editorial section of a local newspaper.
    "This past winter, 200 students from Waymarsh State College traveled to the state capitol building to protest against proposed cuts in funding for various state college programs. The other 12,000 Waymarsh students evidently weren’t so concerned about their education: they either stayed on campus or left for winter break. Since the group who did not protest is far more numerous, it is more representative of the state’s college students than are the protesters. Therefore the state legislature need not heed the appeals of the protesting students."
    Discuss how well reasoned . . . Etc.

    Analysis

    The article concludes that the state legislature does not need to consider the positions of protesting students. To support this argument, the author cited that only 200 of the 12,000 students traveled to the state capitol to claim their concerns on the proposed cuts in college programs. The remaining students do not participate in this protest and therefore are not interested in this issue. The argument is not convincing for the following two reasons.

    First, the author assumes that because less than one-tenth of the students participated in the protest, only less than one-tenth of the students proposed against the funding cut programs. This assumption is questionable since if the protesting students were selected randomly from the entire student body, their views would correctly reflect the views of the entire college. Without further information regarding the way by which the protesting students were selected, it is questionable to conclude that their opinions fail to reflect the opinions of their colleagues.

    Second, the author points out that the other 12,000 students stayed on campus or left for winter break and concludes that they are not concerned on fund-cutting program. The line of reasoning is not solid because there may be other reasons that they did not participate. For example, they probably think their opinions will be expressed by the protesting students. Without addressing why the remaining 12,000 students did not attend the campaign, the argument that the remaining 12,000 students are not concerned about their education is unacceptable.

    In conclusion, the argument is neither sound nor convincing. To make it logically acceptable, the author should point out what is the point of view of the remaining 12,000 students.

  • 14.An Argument

    The following appeared in the editorial section of a local newspaper:

    "In the first four years that Montoya has served as mayor of the city of San Perdito, the population has decreased and the unemployment rate has increased. Two businesses have closed for each new business that has opened. Under Varro, who served as mayor for four years before Montoya, the unemployment rate decreased and the population increased. Clearly, the residents of San Perdito would be best served if they voted Montoya out of office and reelected Varro."

    Discuss how well reasoned you find this argument. In your discussion be sure to analyze the line of reasoning and the use of evidence in the argument. For example, you may need to consider what questionable assumptions underlie the thinking and what alternative explanations or counterexamples might weaken the conclusion. You can also discuss what sort of evidence would strengthen or refute the argument, what changes in the argument would make it more logically sound, and what, if anything, would help you better evaluate its conclusion.

    Analysis

    The argument claims that the residents of San Pedito would be better served if they vote the current mayor Montoya out of office and reelect the former mayor. The evidence cited to support this conclusion is that the unemployment rate has increased and the population has decreased during the first four years of Montoya’s ruling. To further support the argument, the author also stated that the situation of unemployment and population was reversed when Varro served as mayor. However, I think that this argument is not convincing, the reasons are as following:

    First of all, the author readily assumes that the current mayor's governing is the main cause of decreasing population and increasing unemployment rate. There’re some other factors that should be taken into consideration. For example, the economic recession might have occurred in the past four years so that it seriously impacted the local business in San Pedito. Also, it could be possible that a natural disaster has happened, such as flooding and tornado, causing some mortalities and forcing residents to leave.

    Moreover, the unemployment rate and population are not the only indicators of the mayor’s performance. More indicators should be involved to evaluate a mayor’s ability, such as the neighborhood security and education quality. The two mayors may have focused on different aspects while they are serving the residents. It’s unfair to compare the performance between those two person solely based on the unemployment rate and population.

    Finally, the argument cites that two businesses have closed for each new business that has opened during the four years of Montoya serving as mayor and it readily considers that this is a bad sign for the city. However, this could be a good sign instead. Nowadays, with the newly developed technology and advanced management measure, the businesses are able to increase the efficiency in production and operation, so it's not surprising that one business could now achieve what two business could do before.

    In sum, it would be prudent for the city of San perdito to replace the mayor based on the evidences provided. The conclusion would be more reasonable if the author could prove that eliminate the other factors causing the increased unemployment rate and decreased population, and evaluate the mayor’s performance with more indicators.

  • 15.An Argument

    The following appeared as part of a promotional campaign to sell advertising space in the Daily Gazette to grocery stores in the Marston area.

    Advertising the reduced price of selected grocery items in the Daily Gazette will help you increase your sales. Consider the results of a study conducted last month. Thirty sale items from a store in downtown Marston were advertised in the Gazette for four days. Each time one or more of the 30 items was purchased, clerks asked whether the shopper head read the ad. Two-thirds of the 20 shoppers asked answered in the affirmative. Furthermore, more than half the customers who answered the affirmative spent over $100 at the store.

    Discuss how well reasoned you find this argument. In your discussion be sure to analyze the line of reasoning and the use of evidence in the argument. For example, you may need to consider what questionable assumptions underline the thinking and what alternative explanations or counterexamples might weaken the conclusion. You can also discuss what sort of evidence would strengthen or refute the argument, what changes in the argument would make it more logically sound and what, if anything, would help you better evaluate in conclusion.

    Analysis

    The conclusion of this argument is that advertising the reduced price of selected grocery items in the Daily Gazette will result in increased sales overall.

    To support this claim, the author cites an informal poll conducted by sales clerks when customers purchased advertised items. When each time one or more of the advertised items was sold, the clerks asked whether the customer had read the ad. It turned out that two-thirds of 200 shoppers questioned said that they had read the ad. In addition, of those who reported reading the ad, more than half spent over $100 in the store.

    This argument is unconvincing for two reasons.

    To begin with, the author's line of reasoning is that the advertisement was the cause of the purchase of the sale items. However, while the poll establishes a correlation between reading the ad and purchasing sale items and also indicates a correlation, though less significantly, between reading the ad and buying non-sale items, it does not establish a general causal relationship between these events.

    To establish this relationship, other factors that could bring about this result must be considered and eliminated. For example, if the four days during which the poll was conducted preceded THANKSGIVING and the advertised items were traditional foodstuffs associated with this holiday, then the results of the poll would be extremely biased and unreliable.

    Moreover, the author assumes that the poll indicates that advertising certain sale items will cause a general increase in sales. But the poll does not even address the issue of increased overall sales. It informs us mainly that, of the people who purchased sales items, more had read the ad than not.

    A much clearer indicator of the ad's effectiveness would be a comparison of overall sales on days the ad ran with overall sales on otherwise similar days when the ad did not run.

    In sum, this argument is defective mainly because the poll does not support the conclusion that sales in general will increase when reduced-price products are advertised in the Daily Gazette. To strengthen the argument, the author must, at the very least, provide comparisons of overall sales reports as described above.

  • 16.An Argument

    The following appeared as part of a campaign to sell advertising time on a local radio station to local businesses.

    The Cumquat Cafe began advertising on our local radio station this year and was delighted to see its business increase by 10 percent over last year's totals. Their success shows you how you can use radio advertising to make your business more profitable.

    Discuss how well reasoned you find this argument. In your discussion be sure to analyze the line of reasoning and the use of evidence in the argument. For example, you may need to consider what questionable assumptions underline the thinking and what alternative explanations or counterexamples might weaken the conclusion. You can also discuss what sort of evidence would strengthen or refute the argument, what changes in the argument would make it more logically sound and what, if anything, would help you better evaluate in conclusion.

    Analysis

    In an attempt to sell radio advertising time, this ad claims that radio advertising will make businesses more profitable. The evidence cited is a ten percent increase in business that the Cumquat Cafe has experienced in the year during which it advertised on the local radio station.

    This argument is unconvincing because two questionable assumptions must be made for the stated evidence to support the author's conclusion.

    The first assumption is that radio advertising alone has caused the increase in business at the Cumquat Cafe. This assumption is questionable because it overlooks a number of other factors that might have contributed to the Cumquat's success. For example, the Cumquat might have changed owners or chefs. It might have launched a coupon ad campaign in the local print media. Or it might have changed or updated the menu.

    Yet another possibility is that a local competitor went out of business. These are just a few of the factors that could help explain the Cumquat's growth. Because the author fails to eliminate these possibilities, the assumption in question need not be accepted.

    Even if it is granted that radio advertising is responsible for the Cumquat's success, another assumption must be made before we can conclude that radio advertising will result in increased profits for businesses in general. We must also assume that what is true of the Cumquat will likewise be true of most other businesses.

    But there are all kinds of important differences between cafes and other businesses that could affect how radio audiences react to their advertising. We cannot safely assume that, because a small restaurant has benefited from radio advertising, any and all local businesses will similarly benefit.

    In conclusion, it would be imprudent for a business to invest in radio advertising solely on the basis of the evidence presented. To strengthen the conclusion, it must be established that radio advertising was the principal cause of increased business at the Cumquat.

    Once this is shown, it must be determined that the business in question is sufficiently similar to the Cumquat and so can expect similar returns from investment in radio ad time.

  • 17.An Argument

    The following appeared as part of a newspaper editorial.

    Two years ago Nova High School began to use interactive computer instruction in three academic subjects. The school dropout rate declined immediately and last year's graduates have reported some impressive achievements in college. In future budgets the school board should use a greater portion of the available funds to buy more computers and all schools in the district should adopt interactive computer instruction throughout the curriculum.

    Discuss how well reasoned you find this argument. In your discussion be sure to analyze the line of reasoning and the use of evidence in the argument. For example, you may need to consider what questionable assumptions underline the thinking and what alternative explanations or counterexamples might weaken the conclusion. You can also discuss what sort of evidence would strengthen or refute the argument, what changes in the argument would make it more logically sound and what, if anything, would help you better evaluate in conclusion.

    Analysis

    The editorial recommends that the school board of Nova High spend a greater portion of available funds on the purchase of additional computers and adopt interactive computer instruction throughout the curriculum.

    Two reasons are offered in support of this recommendation.

    First, the introduction of interactive computer instruction in three academic subjects was immediately followed by a decline in the school dropout rate. Second, impressive achievements in college were experienced by last year's graduates.

    This argument is unconvincing for two reasons.

    To begin with, this argument is a classic instance of "after this, therefore because of this" reasoning. The mere fact that the introduction of interactive computer instruction preceded the impressive performance of recent graduates and the decline in the dropout rate is insufficient to conclude that it was the cause of these events. Many other factors could bring about these same results.

    For example, the school may have implemented counselling and training programs that better meet the needs of students who might otherwise leave school to take jobs. In addition, the school may have introduced programs to better prepare students for college.

    Secondly, the author assumes that the impressive achievements of last year's graduates bear some relation to the introduction of interactive computer instruction at Nova High. However, no evidence is offered to support this assumption.

    Lacking evidence that links the achievements of the recent graduates to the interactive instruction, it is presumptuous to suggest that the computer instruction was in some way responsible for the students' impressive performance.

    In conclusion, the recommendation that Nova High spend a greater portion of available funds on the purchase of additional computers and adopt interactive computer instruction throughout the curriculum is ill-founded. To strengthen this recommendation the author will have to demonstrate that the decline in the dropout rate and the impressive performance of recent graduates came about mainly as a result of theuse of computer-interactive instruction. All that has been shown so far is an accidental correlation between these events.

  • 18.An Argument

    The following appeared as part of advertisement for Adams, who is seeking reelection as governor

    "Reelect Adams , and you will be voting for proven leadership in improving the state's economy. Over the past year alone, 70 percent of the state's workers have had increases in their wages , 5000 new jobs have been created, and six corporations have located their headquearters here. Most of the respondents in a recent poll said they believed that the economy is likely to continue to improve if Adams is reelcted. Adams's oponent , Zebulon, would lead our state in the wrong direction, because Zebulon disagrees with many of Adams's economic policies."

    Discuss how well reasoned....etc.

    Analysis

    The author concludes that Adams is a good choice for the position of Governor because Adam's policy has contributed to increase in wages of workers and more new jobs created.Since Zebulon disagees with many of Adam's economic policy , he would lead the state in wrong direction. However, the conclusion relies on several assumptions which have no clear evidence.

    Firstly, the author assumes that the creation of new jobs, increase in wages are only because of Adams's economic policy. In fact, there could be many other factors which lead to creation of more jobs and increment in wages of workers.Even after this improved situation in state's economy , the question which author fails to answer is : Is this sufficient? It might be possible that the economic conditions were worse in past and it didnt improve much due to Adams's economic policy.The author also didnt say anything about the policy adopted by Zebulon. It's possible that Zebulon's policy would yield better results in the improvement of economic conditions which would have otherwise been limited due to less effective policies adopted by Adams.The augument would be strenghtened if the author provided evidence that Zebulon's economic policy would result in poor advancement as compared to Adams's policy.

    In additon, the author assumes that the respondents in a recent poll favoring Adams's reelection are true picture of entire population of the state. In fact the author doesn't mention whether or not the respondents of recent poll are the people who are educated and aware of the economic policies of both Adams and Zebulon. It might be possible that the respondents are Adams's supporters or friends and they are in favor of Adam's reelection because of their own vested interests.The argument would be strenghtened considerably if the author provided evidence that the respondents in survey are the people who are not biased, have state's economic interest as priority and they reflect the true sample of entire state' populations.

    Finally, the author assumes that even after reelection Adams would contribute to the improvement of economic condition of state.The author provides no evidence to indiacte that this is the case.As the argument stands its highly possible that the economic condition is at its best at present and Adams's policy which Zebulon is opposing wont be effective in further improvement .Only if the suthor provides evidence that Adams's policy would lead the state to more better advancement - does the argument hold.

    Because the argument leaves out several key issue, it is not persuasive.If the author included the argument above , it would have been more convincing.

  • 19.An Argument

    The following appeared as part of an article in the education section of a Waymarsh City newspaper.

    "Throughout the last two decades, those who earned graduate degrees found it very difficult to get jobs teaching their academic specialties at the college level. Those with graduate degrees from Waymarsh University had an especially hard time finding such jobs. But better times are coming in the next decade for all academic job seekers, including those from Waymarsh. Demographic trends indicate that an increasing number of people will be reaching college age over the next ten years; consequently, we can expect that the job market will improve dramatically for people seeking college-level teaching positions in their fields."

    Analysis

    The author concludes that graduates from Waymarsh University would be easily recruited for the jobs teaching their academic professions at the college level. To substantiate the conclusion, the author points out the demographic forecasts which show that the next ten years the number of college students will increase, and thus more job offers would be available. This argument, however, fails to be persuasive for the author's questionable assumptions, statistical error and ultimately illogical causality.

    Most conspicuously, the author's line of reasoning is that the demographic forecasts are accurate, and certainly remain unchanged during the period of ten years. Actually, this assumption is not necessarily valid. Many influential factors can bring about the fluctuated number of population. For example, SARS epidemic disperses globally next two years among major cities. Natural disaster annihilates some regions of the country. Apparently, any of these crises would have impact on the number of population, as well as that of college students. Hence, the period of ten years is still too far to precisely determine the future trends.

    Also, the author does not address the validity of the prediction of demographic trends. The given trends may be biased. If the demographic survey is conducted by Waymarsh University's partisans, or even Waymarsh University itself, then the claim that the number of college age students would increase in the next decade is not cogent. Presumably, Waymarsh University intends to constitute such survey especially to negate the notorious claim that their graduates are mostly unemployed due to low standard, and to attract more students to enroll to the Waymarsh University's academic program. Obviously, such a result would be unreliable, and cannot longer be used as future prediction.

    Finally, this argument is based on the flawed assumption that the number of Waymarsh graduates is now outnumbered that of college students. Consequently, those graduates cannot attain teaching positions in college. However, this is a dubious claim. No evidences does the author use to support this assumption. Possibly, the actual cause of unemployment of Waymarsh graduates is that those graduates are not qualified. The academic program of Waymarsh University may be not standardized, and thus fails to produce qualified graduates. Obviously, the author does not clearly elucidate the root cause of the unemployed graduates issue; therefore, the author's argument is incomplete.

    In summary, this argument is unwarranted. To strengthen the conclusion, the author would have to provide additional evidence that the number of college students would definitely increase in the next ten years. Nothing can change this trend. Moreover, the author has to prove that the forecasts are reliable and not partial. Most importantly, the actual reasons why Waymarsh graduates are difficult to seek college-level instructing jobs should be given. Without such evidences mentioned above, this argument remains logically unconvincing, and fails to impress the readers.

  • 20.An Argument

    The following appeared in an article in a consumer-products magazine:

    "Two of today’s best-selling brands of full-strength prescription medication for the relief of excess stomach acid, AcidEase and Pepticaid, are now available in milder nonprescription forms. Doctors have written 76 million more prescriptions for full-strength Acid-Ease than for full-strength Pepticaid. So people who need an effective but milder nonprescription medication for the relief of excess stomach acid should choose Acid-Ease."

    Analysis

    In the preceding statement the author makes the claim that Acid-Ease is more effective than Pepticaid since doctors have written more prescriptions for the stronger Acid-Ease over Peptiacid. Though the argument may very well have its merit, it is based on several questionable premises and assumptions, and based on the evidence provided, it is certainly not conclusive.

    The primary issue lies in the author’s unsubstantiated premises. First, the author claims that the doctors have written 76 million more prescriptions for the full-strength Acid-Ease, however he does not mention the total number of prescriptions. If, for example the total number of prescriptions were 10 billion, the difference- 76 million is a very small number. Also, he does not mention source of the facts and the duration of the survey. If for example, the duration was last 10 years and Pepticaid was available in the market only in the last 2 years, then this number is questionable. The author’s premises, the basis of the argument, lack evidence.

    In addition to this, the author makes several assumptions in the above argument. First the author assumes that more written prescriptions must certainly mean most effective medicine. It could be that the company that manufactures AcidEase treated the doctors very well, by providing the doctors and their families with all-paid expense trips. The doctors could, hence be biased in their prescription of medication. Also, even if AcidEase was effective. It need not be that the milder AcidEase also be effective. There is a possibility that Pepticaid may be more effective in the case of milder medications. The author weakens the argument by making these assumptions.

    While in the above argument has several key issues, it cannot be said that the argument is completely off base. The author could have provided more evidence for the above premises by giving more details of the source of facts and by providing more details like the time since both medications are available in market. Though there are several flaws in the argument, with more research and clarification, the author can significantly improve the argument.

    In sum, the author’s illogical argument is based on many unsubstantiated premises and questionable assumptions that render the conclusion invalid. If the author truly hopes to change the readers’ minds he needs to fix the flaws in logic, clarify his assumptions and provide more evidence. Without these, the poorly structured argument would convince few people.

  • 21.An Argument

    The following is an excerpt from a memo written by the head of a governmental department:

    "Neither stronger ethics regulations nor stronger enforcement mechanisms are necessary to ensure ethical behavior by companies doing business with this department. We already have a code of ethics that companies doing business with this department are urged to abide by, and virtually all of these companies have agreed to follow it. We also know that the code is relevant to the current business environment because it was approved within the last year, and in direct response to specific violations committed by companies with which we were then working—not in abstract anticipation of potential violations, as so many such codes are."

    Discuss how well reasoned you find this argument. In your discussion be sure to analyze the line of reasoning and the use of evidence in the argument. For example, you may need to consider what questionable assumptions underlie the thinking and what alternative explanations or counterexamples might weaken the conclusion. You can also discuss what sort of evidence would strengthen or refute the argument, what changes in the argument would make it more logically sound, and what, if anything, would help you better evaluate its conclusion.

    Analysis

    The argument states that last year a specific governmental department approved a code of ethics for its business partners after specific violations took place. All companies that work with the department are asked to comply to the code and most companies agreed to do so. Since there is a functioning ethics code in place, there is no need for a legal regulation or additional enforcement mechanisms.

    At first glance, this argument may seem somewhat convincing, though closer examination reveals lack of evidence, ill-founded assumptions and poor reasoning. For example, the argument assumes the effectiveness of the code without providing further evidence. Additionally, some companies clearly do not commit to the code. And finally, an independent body of oversight may be required even if the code is effective. Consequently, this argument is rather weak, dubious and unconvincing.

    First, the argument relies on the assumption that because the code is relevant to the business environment and was approved recently, it is effective in preventing ethics conflicts. On the one hand, it is reasonable to believe that relevance, concreteness and actuality is required for an effective code. On the other hand, these characteristics are necessary pre-conditions, not proof of its effectiveness. Clearly, there is no reason to believe that the code can be effective. To allow for a better evaluation of the argument, the author needs to provide more supporting evidence for his belief.

    Second, the argument states that vitually all companies have agreed to follow the code. However, just one misstep by a single corporation can be a serious violation of ethics. Also if the companies agreed to follow the code, how is it ensured that they actually do so? Since there was a violation year, what changes have been made to ensure that this will not happen again? Without convincing answers to these questions, this seems more like a wishful thinking rather than substantive evidence. The argument could be considerably strengthened if it explained how the ethics compliance is ensured besides a simple agreement to a code.

    Third, the argument assumes that the use of the code of ethics makes stronger ethics regulations or stronger enforcement mechanisms unnecessary. Yet, there are many examples where independent oversight was necessary to prevent a moral dilemma. To illustrate, consider Volkswagen and the diesel scandal. Even though a code of conduct was in place, the engineers of Volkswagen manipulated the software of the car in order to show reduced emissions during testing compared to realistic driving. This manipulation took place under the eye of the leadership who should have been responsible for ensuring the application of the code of conduct. However, there was no independent body to oversee the compliance. Consequently, the author fails to make a convincing case for his conclusion. To give the argument more credibility, the author must clearly state why he believes why a independent oversight is not necessary.

    In conclusion, the argument is not completely well-reasoned as it stands. To make a more convincing case, the author should prove more information on what the contents of the ethics code are and how exactly compliance to the code is ensured.

  • 22.An Argument

    The following appeared as part of an article in the travel section of a newspaper.

    Over the past decade the restaurant industry in the country of Spiessa has experienced unprecedented growth. This surge can be expected to continue in the coming years fuelled by recent social changes. Personal incomes are rising. More leisure time is available. Single-person house-holds are more common and people have a greater interest in gourmet food as evidenced by a proliferation of publications on the subject.

    Discuss how well reasoned you find this argument. In your discussion be sure to analyze the line of reasoning and the use of evidence in the argument. For example, you may need to consider what questionable assumptions underline the thinking and what alternative explanations or counterexamples might weaken the conclusion. You can also discuss what sort of evidence would strengthen or refute the argument, what changes in the argument would make it more logically sound and what, if anything, would help you better evaluate in conclusion.

    Analysis

    Recent social changes in the country of Spiessa lead the author to predict a continued surge in growth of that country's restaurant industry. Rising personal incomes, additional leisure time, an increase in single person households and greater interest in gourmet food are cited as the main reasons for this optimistic outlook. All of these factors are indeed relevant to growth in the restaurant industry. So the prediction appears reasonable on its face. However, three questionable assumptions operative in this argument bear close examination.

    The first dubious assumption is that the supply of restaurants in Spiessa will continue to grow at the same rate as in the recent past. However, even in the most favourable conditions and the best of economic times there are just so many restaurants that a given population can accommodate and sustain. It is possible that the demand for restaurants has already been met by the unprecedented growth of the past decade, in which case the recent social changes will have little impact on the growth of the restaurant industry.

    A second assumption is that the economic and social circumstances cited by the author will actually result in more people eating out at restaurants. This assumption is unwarranted, however. For example, increased leisure time may just as likely result in more people spending more time cooking gourmet meals in their own homes.

    Also, single people may actually be more likely than married people to eat at home than to go out for meals.

    Finally, people may choose to spend their additional income in other ways - on expensive cars, travel or larger homes.

    A third poor assumption is that, even assuming people in Spiessa will choose to spend more time and money eating out, no extrinsic factors will stifle this demand. This assumption is unwarranted. Any number of extrinsic factors - such as a downturn in the general economy or significant layoffs at Spiessa's largest businesses - may stall the current restaurant surge.

    Moreover, the argument fails to specify the social changes that have led to the current economic boom. If it turns out these changes are politically driven, then the surge may very well reverse if political power changes hands.

    In conclusion, this argument unfairly assumes a predictable future course for both supply and demand.

    To strengthen the argument, the author must at the very least show that demand for new restaurants has not yet been exhausted, that Spiessa can accommodate new restaurants well into the future and that the people of Spiessa actually want to eat out more.

  • 23.An Argument

    The following appeared in an article in a health and fitness magazine.

    Laboratory studies show that Saluda Natural Spring Water contains several of the minerals necessary for good health and it is completely free of bacteria. Residents of Saluda, the small town where the water is bottled, are hospitalized less frequently than the national average. Even though Saluda Natural Spring Water may be seen expensive, drinking it instead of tap water is a wise investment in good health.

    Discuss how well reasoned you find this argument. In your discussion be sure to analyze the line of reasoning and the use of evidence in the argument. For example, you may need to consider what questionable assumptions underline the thinking and what alternative explanations or counterexamples might weaken the conclusion. You can also discuss what sort of evidence would strengthen or refute the argument, what changes in the argument would make it more logically sound and what, if anything, would help you better evaluate in conclusion.

    Analysis

    In this argument the author concludes that drinking Saluda Natural Spring Water (SNSW) is preferable to drinking tap water.

    Three reasons are offered in support of this conclusion. SNSW contains several of the minerals necessary for good health. It is completely free of bacteria. And residents of Saluda - the town where it is bottled - are hospitalized less frequently than the national average.

    This argument is unconvincing because it relies on a variety of dubious assumptions.

    The first questionable assumption underlying this argument that tap water does not contain the minerals in question and is not completely free of bacteria. This assumption is not supported in the argument. If tap water is found to contain the same minerals and to be free of bacteria, the author's conclusion is substantially undermined.

    A second assumption of the argument is that the water residents of Saluda drink is the same as SNSW. Lacking evidence to the contrary, it is possible that Saluda is not the source of the bottled water but is merely the place where SNSW is bottled. No evidence is offered in the argument to dispute this possibility.

    Finally, it is assumed without argument that the reason why residents are hospitalized less frequently than the national average is that they drink SNSW. Again, no evidence is offered to support this assumption. Perhaps the residents are hospitalized less frequently because they are younger than the national average, because they are all vegetarians or because they exercise daily. That is, there might be other reasons than the one cited to account for this disparity.

    In conclusion, this is an unconvincing argument. To strengthen the conclusion that SNSW is more healthful than tap water, the author must provide evidence that tap water contains harmful bacteria not found in SNSW.

    Moreover, the author must demonstrate that the residents of Saluda regularly drink the same water as SNSW and that this is why they are hospitalized less frequently than the national average.

  • 24.An Argument

    The following appeared in a memorandum from the head of a human resources department at a major automobile manufacturing company to the company's managers:

    "Studies have found that employees of not-for-profit organizations and charities are often more highly motivated than employees of for-profit corporations to perform well at work when their performance is not being monitored or evaluated. Interviews with employees of not-for-profit organizations suggest that the reason for their greater motivation is the belief that their work helps to improve society. Because they believe in the importance of their work, they have personal reasons to perform well, even when no financial reward is present. Thus, if our corporation began donating a significant portion of its profits to humanitarian causes, our employees’ motivation and productivity would increase substantially and our overall profits would increase as well."

    Discuss how well reasoned . . .etc.

    Analysis

    Charities and not-for-profit organizations often have employees with a higher productivity level than employees of for-profit corporations. Based on this information, the author of the argument above makes an argument that his or her corporation should donate significant amounts of profits to humanitarian efforts, to increase the productivity of employees. States in this way, the argument reveals examples of leap of faith and poor reasoning. Although it presents an interesting point, it is a weak argument, and has several flaws.

    First, the argument jumps to the conclusion that employees of their corporation will begin to act more like employees of a not for profit company if more money is donated to charity. If our corporation donates a significant portion of its profits to humanitarian causes, that doesn’t make it a nonprofit; it just makes it more giving than it was before. Although employees would indirectly be improving society, this differs from a nonprofit where employees are directly improving society. Therefore, the premise that the conclusion is based on would not apply in this case; such a conclusion should be based on a premise applying to for profit corporations. A better study to examine would be one comparing the productivity of employees in comparison to amount donated by the company. An even better study would be a controlled experiment, where the productivity of employee would be measured over time as charitable donations by the company are increased. Such an experiment would reveal that a higher donation by the company results in greater employee productivity, and a quantification of the productivity and profits gained would be useful in the determination of the decision to donate to humanitarian causes.

    In addition, one of the premises behind the argument, which states that the reason behind this increased motivation is the knowledge that the work performed in the nonprofit benefits society, is based on interviews with employees. Even though the employees interviewed may support the premise, a well reasoned argument cannot rely on information obtained from interviews. Interviews are a great way to obtain the views of a few individual workers, but is not the most effective method for a larger sample size, which is necessary to make a formative conclusion. The researchers also did no interviews to employees of for-profit companies; it is very possible that those employees would report a similar stance in an interview setting as well. Instead, studies which relied on anonymous surveys comparing the views of not for profit and for profit companies would be a much more reliable source of information to base such a premise on.

    Lastly, the conclusion is also based on the premise that employees are more motivated to work harder, but says nothing about their actual productivity. Just because employees report that they are working harder does not mean that they are actually working harder. A better way to quantify this would be to measure the productivity of the employees while not telling employees that their productivity is being monitored.

    As demonstrated above, the argument is clearly flawed and is lacking in several areas. However, many other studies could be used to bolster the argument stated. Without this information, the argument remains unsubstantiated, and is open to debate.

  • 25.An Argument

    The following appeared as part of an editorial in an industry newsletter.
    While trucking companies that deliver goods pay only a portion of highway maintenance costs and no property tax on the highways they use, railways spend billions per year maintaining and upgrading their facilities. The government should lower the railroad companies’ property taxes, since sending goods by rail is clearly a more appropriate mode of ground transportation than highway shipping. For one thing, trains consume only a third of the fuel a truck would use to carry the same load, making them a more cost-effective and environmentally sound mode of transport. Furthermore, since rail lines already exist, increases in rail traffic would not require building new lines at the expense of taxpaying citizens."

    Analysis

    The argument claims that the government should lower the railroad companies’ since sending goods by rail is a more appropriate way of ground transportation than highway shipping. To support this claim the argument has several reasons: railways spend billions per year for maintaining and upgrading their facilities while trucks pay only a portion of highway maintenance and no property tax on the use of highways, trains consume only a third of the fuel a truck would use to carry the same load making it more cost-effective and environmentally sound mode of transport and increases in rail traffic would not require building new lines since rail lines already exist. Stated in this way the argument manipulates the facts and conveys a distorted view of the situation. The conclusion of the argument relies on assumptions for which there is no clear evidence. Hence, the argument is weak and has several flaws.

    First, the argument readily assumes that railroad companies spend much more money on maintenance of their facilities that the trucking companies do for highway maintenance. The argument is a stretch since this claim in not sufficiently substantiated by the evidence. For example, it is possible that trucking companies pay much more for the maintenance of the highways than the railroad companies for the maintenance of the railroad. Clearly, the argument is unconvincing and need to be strengthened. The argument could have been much clearer if it showed the evidence that the railroad companies spend much more money on maintenance than the trucking companies do.

    Second, the argument states that trucks use three times more fuel that trains do. This is again very weak and unsupported claim as the argument relies on assumption for which there is no any evidence. To illustrate, new types of trucks are much more fuel efficient that the old ones and it is possible that trucks use the same, if not, the smaller amount of fuel per mile. If the argument had provided the evidence that trucks actually use much more fuel compared to trains then the argument would have been a lot more convincing.

    Finally, do railroad companies spend more money on maintenance? Do trains use smaller amount of fuel for transportation that trucks? Without convincing answers to these question, one is left with the impression that the claim is more of a wishful thinking rather than substantive evidence.

    In conclusion, the argument is flawed for the above mentioned reasons and is therefore unconvincing. It could be considerably strengthened if the author clearly mentioned all the relevant facts such as the exact amount of money that the railroad companies spends for maintenance compared to trucking companies and which type of transportation is more fuel efficient. Without this information, the argument remains unsubstantiated and open to debate.

  • 26.An Argument

    The following appeared in the editorial section of a newspaper:
    “As public concern over drug abuse has increased, authorities have become more vigilant in their efforts to prevent illegal drugs from entering the country. Many drug traffickers have consequently switched from marijuana, which is bulky, or heroin, which has a market too small to justify the risk of severe punishment, to cocaine. Thus enforcement efforts have ironically resulted in an observed increase in the illegal use of cocaine.” Discuss how well reasoned . . . etc.

    Analysis

    The conclusion in this argument is that increased vigilance by drug enforcement authorities has resulted in an increase in the illegal use of cocaine. The author reaches this conclusion on the grounds that drug traffickers have responded to increased enforcement efforts by switching from bulkier and riskier drugs to cocaine.
    Presumably, the author's reasoning is that the increased enforcement efforts inadvertently brought about an increase in the supply of cocaine which, in turn, brought about the observed increase in the illegal use of cocaine.
    This line of reasoning is problematic in two important respects.
    In the first place, the author has engaged in Since B happened after A, B was caused by A reasoning. The only reason offered for believing that the increased vigilance caused the increase in cocaine use is the fact that the former preceded the latter. No additional evidence linking the two events is offered in the argument, thus leaving open the possibility that the two events are not causally related but were merely coincidental. This in turn leaves open the possibility that factors other than the one cited are responsible for the increase in cocaine use.
    In the second place, the author assumes that an increase in the supply of cocaine is sufficient to bring about an increase in its use. While this is a tempting assumption, it is a problematic one. The presumption required to substantiate this view is that drug users are not particular about which drugs they use, so that if marijuana and heroin are not available, they will switch to whatever drug is available - cocaine in this case. The assumption does not seem reasonable on its face.
    Marijuana, heroin and cocaine are not alike in their effects on users. Nor are they alike in the manner in which they are ingested or in their addictive properties. The view that drug users' choice of drugs is simply a function of supply overlooks these important differences.
    In conclusion, the author has failed to establish a causal link between increased enforcement efforts and the observed increase in illegal cocaine use. While the enforcement activities may have been a contributing factor, to show a clear causal connection the author must examine and rule out various other factors.

  • 27.An Argument

    The following appeared in a speech delivered by a member of the city council:
    “Twenty years ago, only half of the students who graduated from Einstein High School went on to attend a college or university. Today, two–thirds of the students who graduate from Einstein do so. Clearly, Einstein has improved its educational effectiveness over the past two decades. This improvement has occurred despite the fact that the school’s funding, when adjusted for inflation, is about the same as it was 20 years ago. Therefore, we do not need to make any substantial increase in the school’s funding at this time.” Discuss how well reasoned . . . etc.

    Analysis

    This speaker draws the conclusion that there is no need to substantially increase funding for Einstein High School. To support this conclusion, the speaker claims that Einstein has improved its educational efficiency over the past 20 years, even though funding levels have remained relatively constant. His evidence is that two thirds of Einstein's graduates now go on to college whereas 20 years ago only half of its students did so. This argument suffers from several critical problems.
    To begin with, we must establish the meaning of the vague concept educational efficiency. If the term is synonymous with the rate of graduation to college, then the statistics cited would strongly support the argument.
    But, normally we are interested in something more than just the numbers of students who go on to college from a high school. We also want to know how well the school has prepared students for a successful college experience - that is, whether the school has provided a good secondary education. Thus, for the speaker the term educational efficiency must essentially carry the same meaning as educational quality.
    Given this clarification, one of the speaker's assumptions is that the rate of graduation to college has increased because Einstein is doing a better job of educating its students. However, the fact that more Einstein graduates now go on to college might simply reflect a general trend. And the general trend might have less to do with improved secondary education than with the reality that a college degree is now the standard of entry into most desirable jobs.
    But even if the quality of education at Einstein had improved, would this be a compelling reason to deny Einstein additional funding? I don't think so. It is possible that the school has managed to deliver better education in spite of meagre funding. Teachers may be dipping into their own pockets for supplies and other resources necessary for doing their job well. Perhaps the quality of education at Einstein would improve even more with additional financial support.
    In sum, this argument does not establish the conclusion that additional funding for Einstein is unnecessary. To do so, the speaker would have to provide evidence that the quality of education at Einstein has improved. This could be done by examining student assessment scores or by tracking-students through their college careers to see how many successfully graduate and find jobs. In addition, the speaker would also have to show that Einstein is doing a good job with adequate financial support and not merely in spite of insufficient funding.

  • 28.An Argument

    The following appeared in a memo from the customer service division to the manager of Mammon Savings and Loan:
    “We believe that improved customer service is the best way for us to differentiate ourselves from competitors and attract new customers. We can offer our customers better service by reducing waiting time in teller lines from an average of six minutes to an average of three. By opening for business at 8:30 instead of 9:00, and by remaining open for an additional hour beyond our current closing time, we will be better able to accommodate the busy schedules of our customers. These changes will enhance our bank’s image as the most customer-friendly bank in town and give us the edge over our competition.” Discuss how well reasoned . . . etc.

    Analysis

    The customer-service division of Mammon Savings and Loan recommends that the best way for the bank to attract new customers and differentiate itself from its competitors is to improve its service to customers specifically by reducing waiting time in teller lines, opening for business 30 minutes earlier and closing an hour later. These improvements, it is argued, will give the bank the edge over its competitors and make it appear more customer-friendly.
    For the most part this recommendation is well-reasoned. A few concerns must be addressed, however.
    First, the author assumes that Mammon's competitors are similar to Mammon in all respects other than the ones listed. In fact, Mammon's competitors may be more conveniently located to customers or offer other services or products on more attractive terms than Mammon. If so, Mammon may not gain the edge it seeks merely by enhancing certain services.
    Secondly, the author assumes that the proposed improvements will sufficiently distinguish Mammon from its competitors. This is not necessarily the case. Mammon's competitors may already offer or may plan to offer essentially the same customer-service features as those Mammon proposes for itself. If so, Mammon may not gain the edge it seeks merely by enhancing these services.
    Thirdly, the author assumes that Mammon can offer these improved services without sacrificing any other current features that attract customers. In fact, Mammon may have to cut back other services or offer accounts on less attractive terms, all to compensate for the additional costs associated with the proposed improvements. By rendering its other features less attractive to customers, Mammon may not attain the competitive edge it seeks.
    In conclusion, Mammon’s plan for attracting new customers and differentiating itself from its competitors is only modestly convincing. While improvements in customer service generally tend to enhance competitiveness, it is questionable whether the specific improvements advocated in the recommendation are broad enough to be effective.

  • 29.An Argument

    The following appeared as part of an article in a magazine on lifestyles:
    “Two years ago, City L was listed 14th in an annual survey that ranks cities according to the quality of life that can be enjoyed by those living in them. This information will enable people who are moving to the state in which City L is located to confidently identify one place, at least, where schools are good, housing is affordable, people are friendly, the environment is safe, and the arts flourish.” Discuss how well reasoned . . . etc.

    Analysis

    The author concludes that City L has good schools, affordable housing, friendly people, flourishing arts and a safe environment. To support this claim the author cites an annual survey that ranks cities according to quality of life. Two years ago City L was listed 14th in this survey.
    As it stands this argument is unconvincing.
    First, the author fails to indicate what individual characteristics of cities were used as criteria for the ranking. To the extent that the criteria used in the survey were the same as the features listed by the author in the conclusion, the conclusion would be warranted.
    On the other hand, if the survey employed entirely different criteria - for example, outdoor recreational opportunities or educational achievement levels of adult residents - then the author's conclusion would be wholly unwarranted.
    Secondly, the author provides no indication of how each characteristic was weighted in the ranking. For example, City L may have far and away the most flourishing arts scene among the cities surveyed, but it may have poor schools, unfriendly people and an unsafe environment. The extent to which the survey accurately reflects City L's overall quality of life in this case would depend largely on the relative weight placed on the arts as a factor affecting quality of life.
    Thirdly, the author fails to indicate how many cities were included in the survey. The more cities included in the survey, the stronger the argument and vice versa. For example, if 2,000 cities were surveyed, then City L would rank in the top one percent in terms of quality of life. On the other hand, if only 14 cities were surveyed then City would rank last.
    Finally, the author's conclusion depends on the questionable assumption that the conditions listed by the author have remained unchanged in City L since the survey was conducted two years ago. Admittedly, had ten years elapsed the argument would be even weaker. Yet two years is sufficient time for a significant change in the overall economy, the city's fiscal policies, its financial condition or its political climate. Any of these factors can affect the quality of schools, the extent to which art is flourishing or the cost of housing.
    In conclusion, the author does not adequately support the conclusion. To strengthen the argument, the author must show that the criteria used in the survey were the same as the features listed in the conclusion and were weighted in a way that does not distort the picture in City L.
    To better assess the argument, we would also need more information about the cities included in the survey as well as what changes in City L have occurred during the past two years.

  • 30.An Argument

    The following appeared in a memorandum from a member of a financial management and consulting firm:
    “We have learned from an employee of Windfall, Ltd., that its accounting department, by checking about 10 percent of the last month’s purchasing invoices for errors and inconsistencies, saved the company some $10,000 in overpayments. In order to help our clients increase their net gains, we should advise each of them to institute a policy of checking all purchasing invoices for errors. Such a recommendation could also help us get the Windfall account by demonstrating to Windfall the rigorousness of our methods.” Discuss how well reasoned . . . etc.

    Analysis

    In this argurnent a member of a financial management and consulting firm reasons that since Windfall Ltd. increased its net gains by checking 10 percent of its purchasing invoices for errors, it would be a good idea to advise the firm's clients to institute a policy of checking all purchasing invoices for errors.
    Two potential benefits are foreseen from this recommendation. It could help the firm's clients increase their net gains and it could help the firm land the Windfall account.
    The member's argument is unconvincing for a couple of reasons.
    The main problem with the argument is that the conclusion is based upon insufficient evidence. The fact that some of Windfall's purchasing invoices contained errors might simply be attributable to the sloppy accounting practices of Windfall's suppliers. Thus, rather than indicating a general problem, the invoice errors might simply be indicative of a problem that is specific to Windfall Ltd. In other words, the evidence drawn from Windfall's experience is insufficient to support the conclusion that all purchasing invoices of all companies are subject to similar errors.
    Secondly, the evidence offered in the argument suggests only that companies purchasing from the same suppliers that Windfall purchases from are likely to experience similar problems. If the firm's clients do not purchase from Windfall's suppliers, checking for errors might turn out to be a monumental waste of time.
    In conclusion, the author's argument fails to provide good grounds for instituting the policy of routinely checking purchasing invoices for errors. To strengthen the conclusion the author would have to provide evidence that this is a widespread problem. Specifically, what are required are additional instances of purchasing invoices containing errors that are drawn from various companies.

  • 31.An Argument

    The following appeared in a newspaper editorial:
    “As violence in movies increases, so do crime rates in our cities. To combat this problem we must establish a board to censor certain movies, or we must limit admission to persons over 21 years of age. Apparently our legislators are not concerned about this issue since a bill calling for such actions recently failed to receive a majority vote.” Discuss how well reasoned . . . etc.

    Analysis

    Based upon a correlation between increases in movie violence and crime rates in cities, the author argues that to combat crime in cities we must either censor moves that contain violence or prohibit people who are under 21 years of age from viewing them. The author further argues that because legislators failed to pass a bill calling for these alternatives, they are not concerned with the problem of crime in our cities.
    The author's reasoning is unconvincing, since it suffers from two critical problems.
    To begin with, the author's solution to the problem rests on the claim that portrayals of violence in movies are the cause of crime in the cities. However, the evidence offered is insufficient to support this claim. A mere positive correlation between movie violence and city crime rates does not necessarily prove a causal relationship. In addition, all other prospective causes of city crime such as poverty or unemployment must be ruled out. As it stands, the author's solution to the problem is based upon an oversimplified analysis of the issue.
    Another problem with the argument is that the author's solution assumes that only persons under 21 years of age are adversely affected by movie violence. Ultimately, this means that the author is committed to the view that, for the most part, the perpetrators of crime in cities are juveniles under 21. Lacking evidence to support this view, the author's solution cannot be taken seriously.
    In conclusion, the best explanation of the failure of the bill calling for the actions proposed in this argument is that most legislators were capable of recognizing the simplistic analysis of the problem upon which these actions are based. Rather than providing a demonstration of a lack of concern about this issue, the legislators' votes reveal an understanding of the complexities of this problem and an unwillingness to accept simple solutions.

  • 32.An Argument

    The following appeared in the editorial section of a local newspaper:
    “Commuter use of the new subway train is exceeding the transit company’s projections. However, commuter use of the shuttle buses that transport people to the subway stations is below the projected volume. If the transit company expects commuters to ride the shuttle buses to the subway rather than drive there, it must either reduce the shuttle bus fares or increase the price of parking at the subway stations.” Discuss how well reasoned . . . etc.

    Analysis

    The author concludes that the local transit company must either reduce fares for the shuttle buses that transport people to their subway stations or increase parking fees at the stations.
    The reasons offered to support this conclusion are that commuter use of the subway train is exceeding the transit company's expectations, while commuter use of the shuttle buses is below projected volume.
    This argument is unconvincing because the author oversimplifies the problem and its solutions in a number of ways.
    To begin with, by concluding that the transit company must either reduce shuttle fares or increase parking fees, the author assumes that these are the only available solutions to the problem of limited shuttle use. However, it is possible that other factors such as inconvenient shuttle routing and/or scheduling, safety concerns or an increase in carpools contribute to the problem. If so, adjusting fares or parking fees by themselves might not solve the problem.
    In addition, the author assumes that reducing Shuttle fees and increasing parking fees are mutually exclusive alternatives. However, the author provides no reason for imposing an either/or choice. Adjusting both shuttle fares and parking fees might produce better results. Moreover, if the author is wrong in the assumption that parking fees and shuttle fees are the only possible causes of the problem, then the most effective solution might include a complex of policy changes - for example, in shuttle fares, parking fees, rerouting and rescheduling.
    In conclusion, this argument is weak because the author oversimplifies both the problem and its possible solutions. To strengthen the argument the author must examine all factors that might account for the shuttle's unpopularity. Additionally, the author should consider all possible solutions to determine which combination would bring about the greatest increase in shuttle use.

  • 33.An Argument

    The following was excerpted from the speech of a spokesperson for Synthetic Farm Products, Inc.:
    “Many farmers who invested in the equipment needed to make the switch from synthetic to organic fertilizers and pesticides feel that it would be too expensive to resume synthetic farming at this point. But studies of farmers who switched to organic farming last year indicate that their current crop yields are lower. Hence their purchase of organic farming equipment, a relatively minor investment compared to the losses that would result from continued lower crop yields, cannot justify persisting on an unwise course. And the choice to farm organically is financially unwise, given that it was motivated by environmental rather than economic concerns.” Discuss how well reasoned . . . etc

    Analysis

    This speaker argues that farmers who invested in organic farming equipment should resume synthetic farming because it is financially unwise to continue organic farming. The speaker cites studies showing that farmers who switched to organic farming last year had lower crop yields.
    Based on these studies, the speaker concludes that the relatively inexpensive investment in organic farming equipment cannot justify continuing to farm organically. .
    The speaker also claims that continuing to farm organically is financially unwise because it is motivated by envirormental, not economic, concerns.
    His arguments suffer from three problems.
    One problem with this reasoning involves the vague comparative claim that farmers who switched to organic farming last year had lower crop yields. We are not informed whether the survey compared last year's organic crop yields with yields from previous years or with those from synthetic farms. Moreover, the author provides no evidence about how the survey was conducted. Lacking more information about the survey, we cannot accept the speaker's conclusion.
    Secondly, the speaker assumes that the low crop yields for first-time organic farmers last year are representative of crop yields for organic farmers overall. However, more experienced organic farmers might have had much better crop yields last year. Also, the first-time organic farmers might improve their own crop yields in future years. Moreover, last year's yield may have been unusually low due to poor weather or other factors and thus not indicative of future yields.
    Finally, in asserting that organic farming is financially unwise because it is motivated by environmental instead of economic concerns, the speaker unfairly assumes that a practice cannot be both environmentally and economically beneficial. It is possible that, in the long run, practices that help protect the environment will also result in greater economic benefits. For instance, organic farming methods may better protect soil from depletion of the elements that contribute to healthy crops providing an economic benefit in the long run.
    In conclusion, the speaker's argument is poorly supported and is short-sighted. To better evaluate the argument, we would need more information about the how the survey was conducted especially about the comparison the survey makes. To strengthen the argument, the speaker must present evidence that last years' crop yields from first-time organic farmers are representative of yields among organic farms in general. The author must also provide evidence that environmentally sound practices cannot be economically beneficial as well.

  • 34.An Argument

    The following appeared in a newspaper story giving advice about investments:
    “As overall life expectancy continues to rise, the population of our country is growing increasingly older. For example, more than 20 percent of the residents of one of our more populated regions are now at least 65 years old, and occupancy rates at resort hotels in that region declined significantly during the past six months. Because of these two related trends, a prudent investor would be well advised to sell interest in hotels and invest in hospitals and nursing homes instead.” Discuss how well reasoned . . . etc.

    Analysis

    In this argument prudent investors are advised to stop investing in hotels and invest instead in hospitals and nursing homes. The author cites two related trends - an aging population and a decline in hotel occupancy - as grounds for this advice. To illustrate these trends, the author refers to another region of the country where 20 percent of the population is over 65 years old and where occupancy rates in resort hotels have declined significantly during the past six months.
    This argument is unconvincing in a couple of important respects.
    In the first place, the author provides no evidence to support the claim that the population as a whole is aging and that the hotel occupancy rate in general is declining. The example cited, while suggestive of these trends, is insufficient to warrant their truth because there is no reason to believe that data drawn from this unnamed region is representative of the entire country.
    For example, if the region from which the data was gathered was Florida, it would clearly be unrepresentative. The reason for this is obvious. Florida is populated by a disproportionate number of retired people over 65 years old and is a very popular vacation destination during the winter months. Moreover, resort hotel occupancy in Florida typically declines significantly during the summer months.
    In the second place, the author has provided no evidence to support the claim that the decline in hotel occupancy is related to the aging of the population. The author appears to believe that the decrease in occupancy rates at resort hotels is somehow caused by the increase in the number of people over age 65. However, the example cited by the author establishes only that these two trends are sequential or simultaneous. It does not establish that the decline in hotel occupancy is only due to an increase in the number of people over the age of 65.
    In conclusion, the author's investment advice is not based on sound reasoning. To strengthen the conclusion, the author must show that the trends were not restricted to a particular region of the country. The author must also show that the cause of the decline in hotel occupancy is the increase in the number of people over 65.

  • 35.An Argument

    The following appeared as part of the business plan of an investment and financial consulting firm:
    “Studies suggest that an average coffee drinker’s consumption of coffee increases with age, from age 10 through age 60. Even after age 60, coffee consumption remains high. The average cola drinker’s consumption of cola, however, declines with increasing age. Both of these trends have remained stable for the past 40 years. Given that the number of older adults will significantly increase as the population ages over the next 20 years, it follows that the demand for coffee will increase and the demand for cola will decrease during this period. We should, therefore, consider transferring our investments from Cola Loca to Early Bird Coffee.” Discuss how well reasoned . . . etc.

    Analysis

    In the given argument the author of the business plan has an assumption that the average consumption of coffee increases with age and even after the age of 60 the consumption remains high.However the average cola consumption declines with increasing age.According to the author both the trends have remained stable for past 40 years.So by looking at these figures the author concludes that the older adults with increase significantly over next 20 years, because the population will be aging over 20 years,which means that the demand for coffee will increase and demand for cola will decrease.
    Therefore, the author suggests that the company should consider transferring investements form Cola to Early Bird coffee.
    The argument is flawed for several reasons as the author does not show relevant statstics that the consumption of coffee increases with age and consumption of cola decreases with age.The author should provide enough information for us to know that from which source the information was gathered.It might be possible that the numbers what the author is reffering to is collected form one area or just one city.So that does not assume that it implies to all the cities and all the countries that the consumption of cola will decrease with age and consumption of cofffee will increase with age.
    The author also quotes that the trends have been stable for past 40 years, but agian the author does not cite relevant source that according to which study the trends are shown; is it an international firm that that conducted the study or is it just an local company who had gathered all the data and drawn conclusion.The author also falis to cite that how many people were included in the study.There can be a possiblity that the study is just based on 100 people.Whereas we need more people to give their views.
    Also the author fails to give information that the company is targetting at a local city or many other countries.The author should give proper information so that the reader can know that what is companys target location.There can be a possiblity that the trend that author mentions Increase in consumption of coffee with age and decrease in consumption of cola with age might be relevant to just one city.If the company wishes to expand its business into ather location this trend might not be valid at all.
    The author also falsely assumes that the trend will continue that old people will continue their intake of coffee even after the age of 60,there can be a possiblity that due to health awaress campaings the Older people might consider to leave coffee and switch to some other herbal option.For this reason the authors assumption that this trend will continue that people are aging is not reasonable.
    In order to validate the argument the author must provide relevant statistics to show that how many people were involved in teh survey and where the survey was exactly conducted.Also the author must reveal that the company targets local customers or international customers.Else the company can run into husge lossess, because there is always a possibility that in one city people might reduce their consumption of cola and increase their consumption of coffee with age.But in other cities or countries this trend might not be vaild at all.

  • 36.An Argument

    The following appeared in the editorial section of a West Cambria newspaper:
    “A recent review of the West Cambria volunteer ambulance service revealed a longer average response time to accidents than was reported by a commercial ambulance squad located in East Cambria. In order to provide better patient care for accident victims and to raise revenue for our town by collecting service fees for ambulance use, we should disband our volunteer service and hire a commercial ambulance service.” Discuss how well reasoned . . . etc.

    Analysis

    The author's conclusion in the given argument is that in order to provide better patient care the volunteer ambulance service be disbanded and a new commercial ambulance service be hired. To accomplish the aforementioned goal the author also suggests that a service fees be charged for ambulance use. However, the argument is not convincing for a couple of reasons.
    First, to justify the need for a commercial ambulance service,the author cites a review, which reveals that the average response time of volunteer ambulance service in west cambria is lower than the response time of commercial ambulance service in east cambria. But author fails to consider that the response time being compared is of different regions and factors such as traffic congestion may cause response time of one region to be more than that of the other region. Also by comparing only the response time author might be overlooking other important criterias such as the advanced life saving equipment present on-board the ambulance. It might be the case that the volunteer service ambulance's are more advanced and provide these features which might have more impact on patient care than the response time.
    Second, the author suggests that in order to achieve the goal of using commercial ambulance service, a fee be charged, but the author fails to provide additional support that whether people in west cambria can afford the fee that is going to be charged. It might be the case that people in west cambria cannot actually afford this fee and hence must rely on the volunteer service.
    In light of the above, i feel that argument is not convincing and it can be strengthened if the author can provide more reasons to support his conclusion and address the concerns raised above.

  • 37.An Argument

    The following is part of a business plan being discussed at a board meeting of the Perks Company:
    “It is no longer cost-effective for the Perks Company to continue offering its employees a generous package of benefits and incentives year after year. In periods when national unemployment rates are low, Perks may need to offer such a package in order to attract and keep good employees, but since national unemployment rates are now high, Perks does not need to offer the same benefits and incentives. The money thus saved could be better used to replace the existing plant machinery with more technologically sophisticated equipment, or even to build an additional plant.” Discuss how well reasoned . . . etc.

    Analysis

    The author of Perks Company's business plan recommends that funds currently spent on the employee benefits package be redirected to either upgrade plant machinery or build an additional plant. The author reasons that offering employees a generous package of benefits and incentives year after year is no longer cost-effective given current high unemployment rates and that Perks can attract and keep good employees without such benefits and incentives.
    While this argument has some merit, its line of reasoning requires close examination.
    To begin with, the author relies on the reasoning that it is unnecessary to pay relatively high wages during periods of high unemployment because the market will supply many good employees at lower rates of pay. While this reasoning may be sound in a general sense, the particular industry that Perks is involved in may not be representative of unemployment levels generally. It is possible that relatively few unemployed people have the type of qualifications that match the job openings at Perks. If this is the case, the claim that it is easier now to attract good employees at lower wages is ill- founded.
    Secondly, the argument relies on the assumption that the cost-effectiveness of a wage policy is determined solely by whatever wages a market can currently bear. This assumption overlooks the peripheral costs of reducing or eliminating benefits. For example, employee morale is likely to decline if Perks eliminates benefits. As a result, some employees could become less productive and others might quit. Even if Perks can readily replace those employees, training costs and lower productivity associated with high turnover may outweigh any advantages of redirecting funds to plant construction.
    Moreover, because the recommended reduction in benefits is intended to fund the retrofitting of an entire plant or the building of a new one, the reduction would presumably be a sizable one. Consequently, the turnover costs associated with the reduction might be very high indeed.
    In conclusion, this argument is not convincing, since it unfairly assumes that a broad employment statistic applies fully to one specific industry and since it ignores the disadvantages of implementing the plan.
    Accordingly, I would suspend judgment about the recommendation until the author shows that unemployment in Perks' industry is high and until the author produces a thorough cost-benefit analysis of the proposed plan.

  • 38.An Argument

    The following appeared in an editorial from a magazine produced by an organization dedicated to environmental protection:
    “In order to effectively reduce the amount of environmental damage that industrial manufacturing plants cause, those who manage the plants must be aware of the specific amount and types of damage caused by each of their various manufacturing processes. However, few corporations have enough financial incentive to monitor this information. In order to guarantee that corporations reduce the damage caused by their plants, the federal government should require every corporation to produce detailed annual reports on the environmental impact of their manufacturing process, and the government should impose stiff financial penalties for failure to produce these reports.” Discuss how well reasoned . . .etc.

    Analysis

    The author concludes that the best solution of environmental damage caused by industrial manufacturing plants is to require every company to produce the detailed annual reports on the environmental effect of their manufacturing process. To substantiate this conclusion, the author points out that the awareness of the environmental issue would urge the companies to solve the problem, but many companies do not have enough funds to observe the environmental impact. Therefore, the annual report is the best solution. This argument, however, fails to be persuasive for the author's questionable assumptions and logical error.
    Most conspicuously, the author's line of reasoning is entirely based on the fallacy that few corporations that unsuccessfully inspect and observe the specific amount and types of damage in the first place have potential to produce detailed annual reports per the government's compulsory. Actually, this is a dubious claim. Presumably, those companies earn extremely low profits, and thus cannot respond to the new practice of annual report. They would be inevitably punished by the stiff fine, and suffer even lower profitability. Since the root cause of problem still exists, the financial penalties would aggravate the situation into the endless cycle. If this is the case, then the expectation of reduced pollution amount is unfounded.
    Also, the author foresees the success of the plan on the flawed assumption that recognizing the environmental impacts would lead to the corporations' further actions to untangle the problem. Nevertheless, this statement is not necessarily valid. Even if the companies perceive the consequences from pollution emitted from their manufacturing plants, they perhaps abandon them all. The necessary improvement to reduce the amount of pollution may require the companies to install new machines, to educate and train their workers, and to reorganize the garbage system. The companies may not agree to do so because they have to spend large amount of money. Again, the government cannot solve the issues without the companies' cooperation.
    Finally, even if every company produces the annual reports as the government's requirement, the author fails to address the validity of those detailed annual reports. If the faith on the integrity of those reports is accurate, then such compulsory would be effective. On the contrary, if the companies produce incorrect and fake data, then these reports would not be very helpful.
    In summary, this argument is unwarranted. To strengthen the argument, the author would have to provide additional corroborate that the corporations that do not have adequate incentive to monitor the environmental damage of their plants have enough resources to produce the suggested annual reports to the government. Moreover, the author has to prove that the awareness of environmental issue would certainly bring about the cooperation of every corporation to solve the problem. Most importantly, the more information about the process of checking the annual reports should also be given. Without such evidences mentioned above, this argument remains logically unconvincing, and fails to persuader the readers.

  • 39.An Argument

    The following appeared as part of a plan proposed by an executive of the Easy Credit Company to the president: “The Easy Credit Company would gain an advantage over competing credit card services if we were to donate a portion of the proceeds from the use of our cards to a well-known environmental organization in exchange for the use of its symbol or logo on our card. Since a recent poll shows that a large percentage of the public is concerned about environmental issues, this policy would attract new customers, increase use among existing customers, and enable us to charge interest rates that are higher than the lowest ones available.” Discuss how well reasoned . . . etc.

    Analysis

    Environmental organizations are gaining a lot of popularity of late. Global Warming, Glacier melting and oil spills in oceans are deteriorating our environment. We need some organizations to work for the betterment of our environment. There are those who think that environmental organizations are here for looking after the environmental issues and then there are those who want to use the popularity of environmental organization as a revenue generating vehicle. In the preceding statement, author claims that The Easy Credit Company would gain an advantage over competing credit card services if Easy Credit Company donates a portion of the proceeds from the use of its credit cards to well-known environmental organization in exchange for the use of environmental organization symbol or logo on Easy Credit Company’s Card. Though his claims may well have merits, author presents a poorly reasoned argument based on several questionable premises and assumptions and based solely on evidence (Survey) the author offers; we cannot accept his argument as valid.
    The primary issue with author’s reasoning lies in his unsubstantiated premises. The author makes an assumption that environmental organization will be willing to form collaboration with Easy Credit Company for an undisclosed sum. The author does not give any substantial evidence that environmental company would be willing to work with Easy Credit Card Company and author fails to consider that Environmental organization could already have such type of collaboration with a competitor company of Easy Credit Card Company. The author’s premises, the basis of his argument, lacks any legitimate support and render his conclusion unacceptable.
    In addition author makes several assumptions that remain unproven. The author considers a recent poll as the indicator of people’s concern about environmental issues. But author forgets that this survey might represent a small fraction of people who are aware of environmental concerns. Author make another assumption that seeing the environment organization’s logo on credit card will fetch new users and will motivate the existing users to spend more. But again author fails to give any solid example to prove this assumption. So the author weakens his argument by making assumptions and failing to provide the explications.
    While the author does have several key issues in his argument’s premises and assumptions, that is not to say that entire argument is without base. With the increasing environmental awareness, the author is right in proposing that use of an environmental organizations’ logo will bolster the company’s environment friendly image among the customers. If author can provide some evidence to prove that using environment organization’s logo will bring new customers then author’s argument can be verified. Though there are several key issues with the author’s reasoning at present, with research and clarification, he could improve his argument significantly.
    In sum, the author’s illogical argument is based on unsupported premises and unsubstantiated assumptions that render his conclusion invalid. If the author truly hoes to change his reader’s minds on the issue, he would have to largely restructure his argument, fix the flaws in his logic, clearly explicate his assumptions, and provide evidentiary support. Without these things, his poorly reasoned argument will convince few people.

  • 40.An Argument

    The following appeared as part of a recommendation from the financial planning office to the administration of Fern Valley University:
    “In the past few years, Fern Valley University has suffered from a decline in both enrollments and admissions applications. The reason can be discovered from our students, who most often cite poor teaching and inadequate library resources as their chief sources of dissatisfaction with Fern Valley. Therefore, in order to increase the number of students attending our university, and hence to regain our position as the most prestigious university in the greater Fern Valley metropolitan area, it is necessary to initiate a fund-raising campaign among the alumni that will enable us to expand the range of subjects we teach and to increase the size of our library facilities.” Discuss how well reasoned . . . etc.

    Analysis

    The arguer has asserted that in the past few years Fern Valley University has suffered from a decline in enrolments and admissions applications and the reason being poor teaching and inadequate library resources. Further, the author have asserted that to increase the number of students to their university and to regain the most position of most prestigious university they need to start a fund-raising campaign among the alumni which would allow them to broaden the range of subjects and increase their size of library facilities. But, the argument relies on assumptions and there is no clear evidence provided in the recommendation and is therefore, weak and unconvincing with several major flaws.
    Firstly, there is deterioration in the enrolments and admissions applications in last few years and the major reason which is provided by students of this university is poor teaching and lack of library resources. However, the arguer doesn't provide substantial evidence about how much has been the decline in the enrolment’s and admissions applications. Further, the arguer should have provided more information about how many students were being asked for the reason behind this decline. There are various questions which need to be answered such as: Is the reason behind the decline of enrolment’s poor teaching and inadequate library resources? Or their might be decline in the enrolment’s due to high fees of the Fern valley university whereas it might happen that the university doesn't produce good placements for the students, or it could be possible that the library have resources but which are obsolete and the area being small. So, to bolster the argument the arguer could have provided the survey result of 100 students and provided sufficient reason behind the decline other than the personal views of students.
    Secondly, the arguer also asserts that to increase the number of students attending the university and to regain their position as the prestigious university in their area they would have to initiate a campaign asking their alumni's to raise funds. But would this be sufficient enough to get enough students? Will the university again regain its position for sure? Even if it regains its position as prestigious university would there be enough enrolment’s and admissions applications that year. What would the university of the teachers which have poor teaching skill? Would the alumni raise enough fund require for the increase of library facilities? These are the questions which need to be answered by the arguer to make it convincing for the reader.

  • 41.An Argument

    The following appeared in an article in a college departmental newsletter:
    “Professor Taylor of Jones University is promoting a model of foreign language instruction in which students receive 10 weeks of intensive training, then go abroad to live with families for 10 weeks. The superiority of the model, Professor Taylor contends, is proved by the results of a study in which foreign language tests given to students at 25 other colleges show that first-year foreign language students at Jones speak more fluently after only 10 to 20 weeks in the program than do nine out of 10 foreign language majors elsewhere at the time of their graduation.” Discuss how well reasoned . . . etc.

    Analysis

    The article states that Professor Taylor at Jones University is promoting a model of foreign language instruction where students will spend 10 weeks of intensive training and go abroad and live with families for 10 weeks. This methodology of teaching has shown better results than any other colleges however the article is unconvincing and the argument is built on weak assumptions. Although the article looks affirmative, author has failed to provide substantial information to support his position.
    Firstly, argument suggests that Jones University is providing a model of foreign language instruction where students are given intensive 10 weeks training before sending them abroad to live with families in order to work on that language on a daily basis. Author has failed to consider certain factors that make a course or major superior to others. For example, ten weeks intensive training might be sufficient for a student who is good at grasping the language quickly but on the other hand this is not an effective program for students who are weak in picking up languages. Therefore argument should provide comparison on various foreign language-teaching models available in the city and highlight the significance of this teaching model.
    Secondly, author claims that students who had undergone this training method picked up the language much faster compared to others however the claim is flawed because it relies only on a test score. A test score cannot decide the proficiency of a student while speaking especially when the objective is to become fluent in speaking and not to excel at writing that language. Besides that, argument does not provide any evidence of test results that would help to at least to make a comparison. Absence of these factors makes the argument vague and incomplete.
    Thirdly, based on the thread of the argument, it is understood that the program is having a different structure from the rest like sending students abroad and minimized classroom coaching. Due to these changes in structure, it’s highly possible that tuition fees and related academic fees will be comparatively higher and this would adversely affect to attract students who are budget focused.
    To conclude, the argument is not logical and author did not provide supportive evidences to prove his position. In order to make the argument valid, all the above-mentioned points should be considered to support his premise.

  • 42.An Argument

    The following appeared as part of an article in the business section of a local newspaper:
    “Motorcycle X has been manufactured in the United States for more than 70 years. Although one foreign company has copied the motorcycle and is selling it for less, the company has failed to attract motorcycle X customers—some say because its product lacks the exceptionally loud noise made by motorcycle X. But there must be some other explanation. After all, foreign cars tend to be quieter than similar American-made cars, but they sell at least as well. Also, television advertisements for motorcycle X highlight its durability and sleek lines, not its noisiness, and the ads typically have voice-overs or rock music rather than engine-roar on the sound track.” Discuss how well reasoned . . . etc.

    Analysis

    The author contends that the foreign company's copy of motorcycle X is not able to attract customers because of reasons other than the characteristic loud noise. To justify this, the author gives the example of foreign cars which sell as well as american cars despite being relatively quiet. Further, the author says that noise should not be a factor affecting sales because it is not advertised. However, the author's reasoning has few serious flaws.
    Firstly, the author is using faulty analogy in comparing motorcycles to cars with respect to noise. Cars are preferred to be quiet, unless they are sports cars. On the other hand, motorcycles are typically preferred for their masculine appeal, and loud engine noise might have a significant effect of enhancing that appeal.
    Secondly, the assumption that noise is not a factor affecting sales because it is not advertised is faulty. Engine noise can be unpleasant for viewers when heard in the confinement of their homes, but it would not be the same when heard in the open. Moreover, the author fails to consider that use of rock music shows the preference of loud music for motorcycle riders. Every advertisement is designed according to the preferences of the targeted customers, for example, an advertisement for girl's footwear has some fashion based soundtrack. Similarly, rock music shows preference for loud noise for motorcycle riders.
    The argument could have been strengthened if the author provided evidence that motorcycle riders do not consider the characteristic noise to be a factor that affects their choice. Customers coming to buy motorcycles in a showroom could be surveyed to find out how appealing is the engine noise for them, and whether they would prefer a noisy motorcycle over a quieter one.
    In conclusion, the author fails to convince that motorcycle X sells better than its foreign version for reasons other than loud noise. The argument is based on a faulty analogy, comparing noisiness of cars to that of motorcycles. Further, the author bases his assumption on insufficient evidence that noise is not advertised, therefore not desired. The argument could have been strengthened if the author could provide evidence suggesting that noise is not one of the factors affecting sales of motorcycles.

  • 43.An Argument

    The following appeared in the editorial section of a campus newspaper:
    “Because occupancy rates for campus housing fell during the last academic year, so did housing revenues. To solve the problem, campus housing officials should reduce the number of available housing units, thereby increasing the occupancy rates. Also, to keep students from choosing to live off-campus, housing officials should lower the rents, thereby increasing demand.” Discuss how well reasoned . . . etc.

    Analysis

    The author of this article argues that, to reverse declining revenues from campus housing rentals, campus housing officials should decrease the number of available housing units and reduce rent prices on the units. The author's line of reasoning is that fewer available units will limit supply while lower rents will increase demand, thereby improving overall occupancy rates and that the resulting increase in occupancy rates will, in turn, boost revenues for the campus. This reasoning is unconvincing for several reasons. To begin with, the author assumes that boosting occupancy rates will improve revenues. All other factors remaining unchanged, this would be the case. However, the author proposes reducing both the supply of units and their rental prices. Both of these actions would tend to reduce revenues. The author provides no evidence that the revenue-enhancing effect of a higher occupancy rate will exceed the revenue-decreasing effect of reduced supply and price. Without such evidence, the argument is unconvincing. Secondly, the author assumes that lowering rents will lead to higher revenues by increasing demand. However, it is possible that demand would decrease, depending on the extent of the rent reduction as well as other factors such as overall enrolment and the supply and relative cost of off-campus housing. Moreover, even if demand increases by lowering rents, revenues will not necessarily increase as a result. Other factors, such as maintenance and other costs of providing campus housing units and the reduced supply of rental units might contribute to a net decrease in revenue. Thirdly, in asserting that lowering rental rates will increase demand, the author assumes that current rental rates are causing low demand. However, low demand for student housing could be because of various other factors. For instance, the student housing units may be old and poorly maintained. Perhaps students find the campus housing rules oppressive and therefore prefer to live off-campus. Or perhaps University enrolments are themselves down generally affecting campus housing occupancy. In conclusion, the author of this editorial has not argued effectively for decrease in the number of available campus housing units and a reduction in rental rates for those units. To strengthen the argument, the author must show that a rent reduction will actually increase demand and that the revenue-enhancing effect of greater demand will outweigh the revenue-reducing effect of a smaller supply and of lower rental rates.

  • 44.An Argument

    The following appeared in an Avia Airlines departmental memorandum:
    “On average, 9 out of every 1,000 passengers who traveled on Avia Airlines last year filed a complaint about our baggage-handling procedures. This means that although some 1 percent of our passengers were unhappy with those procedures, the overwhelming majority were quite satisfied with them; thus it would appear that a review of the procedures is not important to our goal of maintaining or increasing the number of Avia’s passengers.” Discuss how well reasoned . . . etc.

    Analysis

    The argument that very low percentage of complaints indicate that there is no need to update procedures in order to increase passengers is not entirely logically convincing, because it ignores certain crucial assumptions. Firstly, the argument assumes that only 9 out of 1000 travelers had a complaint about baggage-handling procedures. Actually, Author’s evidence provide just recorded complaints. It is possible that more passengers have compliant about rules but doesn’t want to rate it to company. Because it takes time to complain about procedures to airlines companies that people are unwilling to prefer to do it. For example, a passenger had a problem with procedures of Avia Airlines and got angry, then he decided to never use same airlines again, rather than to complain it.
    Secondly, the argument never addresses customer satisfaction. In the world, it is entirely necessary issue for companies that internet and its derivatives present many websites related to companies’ services and customers are able to comment their opinion in there. Perhaps, number of complaints is low, but they could comment dissatisfaction about Avia Airlines that leads to drop in travelers. For example, if company never reply complaints of customers, websites like Trip-Advisor which have millions of visitors, would decrease their customer satisfaction points.
    Finally, the argument omits data about number of total complaints in total and in specific regions. Every year, the millions of people fly with plane as well as in Avia Airlines too. According to given details there are 9000 complaints in every million passengers which quiet big number for an airlines. Total number of customers wasn’t provided in author’s conclusion. Additionally, data was an average of total, it could more in some locations such as big cities. In Metropolitans as Istanbul, New-York or Hong Kong, there are millions of passengers use their airport that such big crowds in airports lead to difficulties to acquire their baggage. For Example, Customers who used Istanbul Airport made 100 complaints out of 1000 but this number drop because of low numbers in other locations.
    Thus, the argument is not completely sound.
    The evidence in support of conclusion could be to detailed research about reasons behind of complaints and numbers of them in different locations. Company executives possibly respond every complaint to increase their reputation in industry that they care every one of their customers as special person.
    Ultimately, the argument might have been weakened by written flaws and wrong assumptions in Author’s argument.

  • 45.An Argument

    The following appeared as part of an article in a weekly newsmagazine: “The country of Sacchar can best solve its current trade deficit problem by lowering the price of sugar, its primary export. Such an action would make Sacchar better able to compete for markets with other sugar-exporting countries. The sale of Sacchar’s sugar abroad would increase, and this increase would substantially reduce Sacchar’s trade deficit.” Discuss how well reasoned . . . etc.

    Analysis

    The author of this article argues that the country of Sacchar can best solve its current trade deficit problem by lowering the price of its main export, sugar. The line of reasoning is that this action would make Sacchar more competitive with other sugar-exporting countries, thereby increasing sales of Sacchar's sugar abroad and, in turn, substantially reducing the trade-deficit. This line of reasoning is unconvincing for a couple of reasons. In the first place, this argument is based on an oversimplified analysis of the trade deficit problem Sacchar currently faces. A trade-deficit occurs when a country spends more on imports than it earns from exports. The author's argument relies on the assumption that earnings from imports will remain constant. However, the author provides no evidence that substantiates this assumption. It is possible that revenues from imports will increase dramatically in the near future. If so, the course of action proposed by the author might be unnecessary to solve Sacchar's trade deficit problem. Conversely, it is possible that revenues from imports are likely to decrease dramatically in the near future. To the extent that this is the case, lowering sugar prices may have a negligible countervailing effect depending on the demand for Sacchar's sugar. In the second place, increasing sales by lowering the price of sugar will not yield an increase in income unless the increase in sales is sufficient to overcome the loss in income due to the lower price. This raises three questions the author fails to address. First, will a price decrease in fact stimulate demand? Second, is demand sufficient to meet the increase in supply? Third, can Sacchar increase the sugar production sufficiently to overcome the deficit? In the absence of answers to these questions, we cannot assess the author's proposal. In conclusion, the author provides an incomplete analysis of the problem and, as a result, provides a questionable solution. To better evaluate the proposal, we would need to know how revenues from imports are likely to change in the future. To strengthen the argument, the author must provide evidence that demand is sufficient to meet the proposed increase in supply and that Sacchar has sufficient resources to accommodate the increase.

  • 46.An Argument

    The following appeared as part of an article in a trade publication:
    “Stronger laws are needed to protect new kinds of home-security systems from being copied and sold by imitators. With such protection, manufacturers will naturally invest in the development of new home-security products and production technologies. Without stronger laws, therefore, manufacturers will cut back on investment. From this will follow a corresponding decline not only in product quality and marketability, but also in production efficiency, and thus ultimately a loss of manufacturing jobs in the industry.” Discuss how well reasoned . . . etc.

    Analysis

    The author of this article warns that, in order to prevent an eventual loss of manufacturing jobs within the industry, stronger laws are needed to protect new kinds of home security systems from being copied and sold imitators.
    This conclusion is based on the following chain of reasoning. With the protection of stronger laws, manufactures will naturally invest in the development of new home security products and production technologies, whereas without such protection, manufacturers will cut back on investment. If manufacturers cut back on investment, then a decline in product quality and marketability as well as in production efficiency will result. This, in turn, will cause the predicted lose of industry jobs.
    This line of reasoning is unconvincing for several reasons.
    To begin with, the author assumes that existing copyright, patent and trade secret laws are inadequate to protect home security system designs. But the author does not explain why these laws don't offer sufficient protection, nor does he offer any evidence to show that this is the case.
    Secondly, the argument depends on the twin assumptions that stronger legal protection will encourage manufacturers to invest in home security-system production while the absence of strong legal protection will have the opposite effect. The author fails to provide any evidence or reasons for accepting these assumptions about cause-and-effect connections between the law and what happens in the marketplace.
    Moreover, both of these assumptions can be challenged. It is possible that stronger protections would not only affect industry investment or jobs overall, but would instead help to determine which companies invested heavily and therefore provided the jobs. For instance, a less-restricted market might foster investment competition among smaller companies whereas stronger legal protections might encourage market domination by fewer, larger companies. .
    In conclusion, I do not find this argument compelling. The author must provide evidence that home security system designs are not being adequately protected by current patent, copyright or trade secret laws. The author must also provide an argument for the assumptions that stronger laws will create more industry jobs overall, while the absence of stronger laws will result in fewer industry jobs.

  • 47.An Argument

    The following appeared in the opinion section of a national newsmagazine:
    “To reverse the deterioration of the postal service, the government should raise the price of postage stamps. This solution will no doubt prove effective, since the price increase will generate larger revenues and will also reduce the volume of mail, thereby eliminating the strain on the existing system and contributing to improved morale.” Discuss how well reasoned . . . etc.

    Analysis

    The writer's argument is flawed on a number of levels. First and foremost is the relationship between revenue and volume. The revenue of a postal system, attributed to stamped delivery is equal to Price x Quantity. The writer assumes that by raising postal rates (price) revenue will increase even though quantity will decrease. Perhaps the writer should instead state that gross margins will increase, this makes more sense since the change in revenue is dependent on both the change in the price and the drop in the volume of sales due to the change in price.
    This leads us to my second point: without more information it is difficult to determine if an increase in price will, in fact, lead to a decrease in volume. For many companies and institutions (especially government institutions such as the IRS), the demand for postal services is inelastic. Meaning that the number of stamps purchased by the IRS is static, based on the number of letters it has to send out, irrespective of the cost of sending those letters out.
    On the other extreme, postage stamps could be extremely sensitive to price, and a small increase in the price could lead to massive declines in demand.
    The writer also states that a decrease in volume will increase morale. Again this is a dubious conclusion; studies have shown that too little work on the job can be worse for morale than too much. In fact a large drop in volume could result in massive layoffs for the postal service which would almost certainly have a negative impact on morale.
    The writer is also assuming that there is a strain on the postal service. I do not believe this statement to be supported by fact. Over the last decade, the internet has taken the place of much of the volume of messages travelling through the postal system; both commercial and personal correspondences. In addition, advances in automation and changes to workplace conditions have made the postal service on the more pleasant and stress free places to work.
    Finally the writer is assuming that a majority of revenue comes from the sales of stamps. These days the postal service captures a much larger portion of their revenue from other services such as courier services.
    Overall, the writer does not seem to have done any research nor does he seem to have any background in basic economic theory. His statement is not provable and is most likely incorrect.

  • 48.An Argument

    The following appeared in an article in the health section of a newspaper:
    “There is a common misconception that university hospitals are better than community or private hospitals. This notion is unfounded, however: the university hospitals in our region employ 15 percent fewer doctors, have a 20 percent lower success rate in treating patients, make far less overall profit, and pay their medical staff considerably less than do private hospitals. Furthermore, many doctors at university hospitals typically divide their time among teaching, conducting research, and treating patients. From this it seems clear that the quality of care at university hospitals is lower than that at other kinds of hospitals.” Discuss how well reasoned . . . etc.

    Analysis

    In this argument the author concludes that university hospitals provide no better care than private or community hospitals do.
    The author bases this conclusion on the following claims about university hospitals. The university hospitals in this region employ 15 percent fewer doctors. They have a 20 percent lower success rate in treating patients. They pay their staffs less money. They make less profit than community hospitals. And they utilize doctors who divide their time between teaching, research and treating patients.
    This argument is unconvincing for several reasons.
    The most egregious reasoning error in the argument is the author's use of evidence pertaining to university hospitals in this region as the basis for a generalization about all university hospitals. The underlying assumption operative in this inference is that university hospitals in this region are representative of all university hospitals. No evidence is offered to support this gratuitous assumption.
    Secondly, the only relevant reason offered in support of the claim that the quality of care is lower in university hospitals than it is at other hospitals is the fact that university hospitals have a lower success rate in treating patients. But this reason is not sufficient to reach the conclusion in question unless it can be shown that the patients treated in both types of hospitals suffered from similar types of maladies. For example, if university hospitals routinely treat patients suffering from rare diseases whereas other hospitals treat only those who suffer from known diseases and illnesses, the difference in success rates would not be indicative of the quality of care received.
    Finally, the author assumes that the number of doctors a hospital employs, its success rate in treating patients, the amount it pays its staff and the profits it earns are all reliable indicators of the quality of care it delivers. No evidence is offered to support this assumption nor is it obvious that any of these factors is linked to the quality of care delivered to patients.
    Moreover, the fact that doctors in university hospitals divide their time among many tasks fails to demonstrate that they do a poorer job of treating patients than doctors at other kinds of hospitals. In fact, it is highly likely that they do a better job because they are more knowledgeable than other doctors due to their teaching and research experience.
    In conclusion, the author's argument is unconvincing. To strengthen the argument, the author will have to demonstrate that university hospitals in this region are representative of all university hospitals as well as to establish a causal link between the various factors cited and the quality of care delivered to patients.

  • 49.An Argument

    The following is part of a business plan created by the management of the Megamart grocery store:
    “Our total sales have increased this year by 20 percent since we added a pharmacy section to our grocery store. Clearly, the customer’s main concern is the convenience afforded by one-stop shopping. The surest way to increase our profits over the next couple of years, therefore, is to add a clothing department along with an automotive supplies and repair shop. We should also plan to continue adding new departments and services, such as a restaurant and a garden shop, in subsequent years. Being the only store in the area that offers such a range of services will give us a competitive advantage over other local stores.” Discuss how well reasoned . . . etc.

    Analysis

    The management of the Megamart grocery store concludes that adding new departments and services is the surest way to increase profits over the next couple of years. They are led to this conclusion because of a 20 percent increase in total sales which was realized after the addition of a pharmacy section to the grocery store.
    On the basis of this experience, they conclude that the convenience of one-stop shopping is the main concern of their customers.
    The management's argument is faulty in several respects.
    In the first place, the management assumes that the increase in total sales was due to the addition of the pharmacy section. However, the only evidence offered to support this conclusion is the fact that the addition of the pharmacy preceded the increase in sales. But the mere fact that the pharmacy section was added before the increase occurred is insufficient grounds to conclude that it was solely responsible for the increase. Many other factors could have brought about this same result. Lacking a detailed analysis of the source of the sales increase, it would be wrong to attribute the increase only to the addition of the pharmacy section.
    In the second place, even if it were the case, this fact alone is insufficient to support the claim that adding additional departments will increase sales even further. It is quite possible that the addition of the pharmacy section increased sales simply because there was no other pharmacy in the vicinity. The additional proposed departments and services, on the other hand, might be well represented in the area and their addition might not have a significant impact on the profits of the store.
    In other words, there may be relevant differences between the pharmacy section and the additional proposed sections that preclude them from having a similar effect on the sales of the store.
    In conclusion, the management's argument is not based on conclusive evidence. To strengthen the conclusion, the management must provide additional evidence linking the addition of the pharmacy section to the increase in total sales. It must also show that there are no exceptional reasons for the sales increase due to the pharmacy section that would not apply to the other proposed additions.

  • 50.An Argument

    The following appeared as part of a column in a popular entertainment magazine.
    “The producers of the forthcoming movie 3003 will be most likely to maximize their profits if they are willing to pay Robin Good several million dollars to star in it— even though that amount is far more than any other person involved with the movie will make. After all, Robin has in the past been paid a similar amount to work in several films that were very financially successful.”

    Analysis

    The argument claims that to maximize their profits the producers of the forthcoming movie 3003 should hire Robin Good to act in the movie regardless of his extremely high fees. To support this conclusion the argument claims that Robin has in the past been paid a similar amount to act in several movies that were financially successful. Stated in this way the argument fails to mention several key factor on basis of which it could be evaluated. The conclusion of the argument relies on assumptions for which there is no clear evidence. Hence, the argument is weak and has several flaws.
    First, the argument readily assumes that the producers will maximize their profits if they hire Robin Good to act in their film. This statement is a stretch since the argument presupposes that this is the only possible solution and that no other action would achieve the same result. For example, it is possible that there are other actors that could help the producers achieve the same results for a smaller fee or the marketing team could use their knowledge to attract more people to see the movie. The argument could have been much more clearer if it stated that hiring of Robin Good is the only solution which can result in maximizing the profits.
    Second, the argument claims that several films in which Robin Good acted were very financially successful in spite of his high fees. This is again very weak and unsupported claim since the argument assumes without justification that a background condition remained the same. Just because Robin Good earned a fortune for his former employers in past, it does not have to mean that this will be the case in present. For example, new and younger actors are more popular than Robin Good and people like to watch them in movies or the quality of Robin Good acting is not so high as it used to be. If the argument provided the evidence that nothing has changed regarding the popularity and acting of Robin Good then the argument would have been a lot more convincing.
    Finally, is Robin Good the only actor that can maximize the profits of the movie? Are there any other solutions that can have the same effect? Without convincing answers to this questions one in left with the impression that the claim is more of a wishful thinking rather than a substantive evidence.
    In order to assess the merits of a certain decision, it is essential to have full knowledge of all contributing factors. In this particular case whether the producers could find other solutions in order to maximize their profits and whether Robin Good is still high-quality and popular actor who can attract people to cinemas. Without this information, the argument remains unsubstantiated and open to debate.

  • Share with Friends